You are on page 1of 115

Mathematics

(Question Bank)

Corporate Office
ALLEN CAREER INSTITUTE
“SANKALP”, CP-6, INDRA VIHAR, KOTA-324005
PHONE : +91 - 744 - 2436001, Fax : +91-744-2435003
E-mail: info@allen.ac.in Website: www.allen.ac.in
MATHEMATICS
[STRAIGHT OBJECTIVE TYPE]
tan 
Q.1 The efficiency E of a screw jack is given by E = where  is variable and  is some
tan(  )

constant angle lying in  0,  . The maximum efficiencyis given by
 2
1  cos  1  sin  cos  cos 
(A) (B) (C) (D)
1  cos  1  sin  3(1  sin ) 1  cos 

Q.2 16 Players P1, P2, ...... P16 take part in a tennis tournament. The order of the matches is chosen in
random. Lower suffix player is better than a highersuffix, the better wins. The probabilitythat the eighth
best reaches the semifinals, is
1 3 8
(A) (B) (C) (D) none
8 25 65

Q.3 Let P(x) be the polynomial of least degree with leading coefficient 1 and other coefficients being
 
rational such that P  3  3  3  .......  = 0. Sum of the digits in P(5) is
 
(A) 6 (B) 7 (C) 8 (D) 9

Q.4 cosec 2° + cosec 4° + cosec 172° + cosec 164° + cosec 32° + cosec 116° – cosec 308°
(A) sin127° cosec1° cosec128° (B) sin127° sin1° cosec128°
(C) sin127° sin1° sin128° (D) cosec127° cosec1° cosec128°

Q.5 Two medians of a triangle with unequal sides are 3 and 6 units. Its area is 3 15 sq. units. The length of
the third median is
(A) 3 3 (B) 3 6 (C) 6 3 (D) 6 6


Q.6 If a1 = 0.6 and a3 = 0.074 are respectiely the first and the third terms of a decreasing G.P. then a
r 1
2
r

equals -
8 3 1
(A) 2 (B) (C) (D)
9 8 2

Q.7 Let ABC be a right triangle with length of side AB = 3 and hypotenuse AC = 5.
BD AB
If D is a point on BC such that = , then AD is equal to
DC AC
4 3 3 5 4 5 5 3
(A) (B) (C) (D)
3 2 3 4

FINAL STEP Page # 2


MATHEMATICS
 2  2  2
Q.8 Let Sn = cot–1  3x   + cot–1  6 x   + cot–1 10x   + ............. + n terms,
 x  x  x
where x > 0. If Lim Sn = 1, then x equals
n 


(A) (B) 1 (C) tan 1 (D) cot 1
4

1 2
Q.9 If A and B are two independent events such that P(A) = and P(B) = , then
2 3
P  (A  B) (A  B) ( A  B)  has the value equal to
1 1 1 2
(A) (B) (C) (D)
3 4 2 3

Q.10 For each real number m, the quadratic polynomial y = (m2 + 4) x2 + (m – 2)2 x – 4m + 2 passes
through (a, c), then order of pair (a, c) is
(A) (–1, 2) (B) (1, –2) (C) (1, 2) (D) (–1, –2)

Q.11 Suppose x and y are real numbers such that (x + 5)2 + (y – 12)2 = 196. The minimum value of
(x2 + y2) is
1
(A) 2 (B) 3 (C) 1 (D)
2

1
Q.12 Let x and y be the positive real numbers such that logxy = . The value of the expression
4
x2
logx(xy5) – log y is
y
31 33 35
(A) (B) (C) (D) 8
4 4 4

Q.13 If a(5 sin2 – 11) = – sin2 + 8 sin  – 9 where a,   R, then the maximum value of a, is
(A) 1 (B) 2 (C) 3 (D) 4

Q.14 Lot A consists of 6 good and 4 defective articles. Lot B consists of 7 good and 3 defective articles.
A new lot C is formed by taking 6 articles from lot A and 4 articles from lot B. The probability that an
article chosen at random from the lot C is defective, is
1 2 9 12
(A) (B) (C) (D)
3 5 25 25

Q.15 In ABC, if a = 9, b = 8 and c = x satisfies 3 cos C = 2, then the value of x is


[Note: All symbols used have usual meaning in triangleABC.]
(A) x = 4 (B) x = 5 (C) x = 6 (D) x = 7

Q.16 If the equation | x2 + 4x + 3 | + 2m – mx = 0 has exactly three solutions, then the value of (m +8)
can be
(A) 15 (B)  15 (C) 60 (D)  60
FINAL STEP Page # 3
MATHEMATICS
Q.17 If coefficient of xk and co-efficient of x2k (k  N) in the expansion of
1 + (1 + x) + (1 + x)2 + ...... + (1 + x)16 are equal, then k equals
(A) 3 (B) 4 (C) 5 (D) 6

Q.18

 
If p is a prime number and p  2 then  2  5   (2) is always divisible by
p

p 1

[Note: [x] denotes greatest integer function less than or equal to x.]
(A) p + 1 (B) p (C) 2p (D) 4p

Q.19 In triangle ABC, if r : b + c : a = 2 : 17 : 13, then angle A is equal to


   
(A) (B) (C) (D)
6 4 3 2
[Note: All symbols used have usual meaning in triangleABC.]

Q.20 If the equation 5 arc tan (x2 + x + k) + 3 arc cot(x2 + x + k) = 2, has two distinct solutions, then the
range of k, is
 5  5 5   5
(A)  0, (B)   ,  (C)  ,   (D)   ,
 4   4 4   4 

100 r · 100C r
Q.21  100
C r 1
equals
r 1

(A) 100 (B) 4950 (C) 5050 (D) 5151

Q.22 The center of variable circle x2 + y2 + 2gx + 2fy + c = 0 lies on the line 2x – 2y + 9 = 0 and the variable
circle cuts the circle x2 + y2 = 4 orthogonally. If the variable circle passes through two fixed points (a, b)
and (c, d) where (b < d) then the value of 2b + d is equal to
(A) 3 (B) 4 (C) 5 (D) 6

Q.23 Consider the family of lines (x – y – 6) + (2x + y + 3) = 0 and (x + 2y – 4) + (3x – 2y – 4) = 0. If the


lines of these 2 families are at right angle to each other then the locus of their point of intersection, is
(A) x2 + y2 – 3x + 4y – 3 = 0 (B) x2 + y2 + 3x – 4y – 3 = 0
2 2
(C) x + y – 3x – 4y – 3 = 0 (D) x2 + y2 + 3x + 4y – 3 = 0

Q.24 A function y = f(x) has a second order derivative f ''(x) = 6(x – 1). If its graph passes through the point
(2, 1) and at that point tangent to the graph is y = 3x – 5, then the area bounded by y = f (x), x = 0,
y = 0, is
1 1 1
(A) 1 (B) (C) (D)
2 3 4

  1
a
Q.25 Let a   0,  , then the value of Lim 3  ln 1  tan a tan x  dx is equal to
 2 a 0 a
0

1 1 1
(A) 1 (B) (C) (D)
3 2 6

FINAL STEP Page # 4


MATHEMATICS
Q.26 Consider the circle z  5i  3 and two points z1 and z2 on it such that z1  z 2 , and

arg (z1) = arg(z2) = . A tangent is drawn at z2 to the circle, which cuts the real axis at z3,
3
then | z3 | is equal to
(A) 3 (B) 13 (C) 11 (D) 4

Q.27 Let P(x) be a polynomal of degree 4 such that P(1) = 7 and attains its minimum value 3 at both
x = 2 and x = 3, then the vale of P(5) is
(A) 25 (B) 39 (C) 47 (D) 55

x
Let y = y(x) satisfy the equation y(x) +  y( t ) dt  x . The value of y(e) is equal to
2
Q.28
1
(A) 2e – 2 + e1 – e (B) 2e – 1 + e e
1 – (C) 2e + 2 + e1 – e (D) 2e + 1 + e1 – e

Q.29 Let L1 be a line which passes through the point P (1, 2, – 4) whose direction ratios are proportional to
2, 3 and 6 and L2 is a line which passes through (3, 3, – 5) and has the same directions as that of L1.
d
If the distance between L1 and L2 is , then sum of the digits in d is
7
(A) 11 (B) 14 (C) 15 (D) 17

Q.30 If the complex numbers z1 and z2 are the solutions of the equation z2 – z + 1 = 0, then the value of the


expression E = z14  z13  2z12  2z1  1  2013

 z 42  z32  2z 22  2z 2  1 
, is
2013

(A) 1 (B) 2 (C) – 2 (D) – 2


where  is the imaginary cube root of unity.

Q.31 The equation of a line which is concurrent with lines L1 : x – 2y –1 = 0 and L2 : x + y – 2 = 0 and
tangent to the circle x2 + y2 – 6x + 2y + 6 = 0, is
(A) 3x – 3y = 4 (B) 9x – 3y = 14 (C) 3x – 9y = 2 (D) none

cos x x 1
Q.32 Let g(x) = 2 sin x x2 2x then the equation of normal drawn to g(x) at x = 0, is
tan x x 1
(A) x = 0 (B) y = 0 (C) x = 1 (D) y = 1

Q.33 If (1 + i)100 is expanded and written in the form (a + bi) where a and b are real numbers, then a =
100!
(A) – 250 (B) 250 –
50! · 50!

100!  1 1 
(C) (D) 100!   
(25!) 2 · 50!  50! · 50! 25! · 75! 

Q.34 Let S  (3, 4) and S'  (9, 12) be two foci of an ellipse. If the coordinates of the foot of the perpendicular
from focus S to a tangent of the ellipse is (1, – 4) then the eccentricity of the ellipse is
5 4 5 7
(A) (B) (C) (D)
13 5 7 13
FINAL STEP Page # 5
MATHEMATICS
ln (1 x ) 1


y
(1  tan 2 y) dy
0 1
Q.35 If Lim = , then the value of a, is
x 0 sin x ea
1
(A) (B) 1 (C) 2 (D) 3
2
Q.36 A tangent line l is drawn from the origin to the curve C : y = 2 x  3 . The area of the region bounded
by l, C and the x-axis is
3 2 3
(A) (B) 3 (C) 2 3 (D)
2 3

Q.37 If a  (a1, a2) satisfythe condition that the point of local minima and the point of local maxima is less than
4 and greater than – 2 respectively for the function f(x) = x3 – 3ax2 + 3 (a2 – 1) x + 1, then the value
of (a2 – a1), is
(A) 2 (B) 4 (C) 5 (D) 6

Q.38 Let P be any point on the circle with OP as diameter (O being origin). The points Q, R are on the same
side of the diameter such that POQ = QOR = . Also P, Q, R be complex number z1, z2, z3
 
respectively such that 2 3 z 22 = 2  3 z1 z3. Then  equals
   
(A) (B) (C) (D)
8 12 18 10

Q.39 The ellipse 4x2 + 9y2 = 36 and the hyperbola 4x2 – y2 = 4 have the same foci and they intersect at right
angles then the equation of the circle through the points of intersection of two conics is
(A) x2 + y2 = 5 (B) 5 (x2 + y2) – 3x – 4y = 0
(C) 5 (x2 + y2) + 3x + 4y = 0 (D) x2 + y2 = 25

Q.40 Let k be the number of real solutions of the equation ex + x – 2 = 0 in the interval [0, 1] and n be the
number of real solutions that are not in [0, 1], then
(A) k = 0 and n = 1 (B) k = 1 and n = 0 (C) k = n = 1 (D) k = n = 0

cot 1 
 tan x 
Q.41 For R, the range of the function f() = 1  tan x  cot x  dx is equal to

tan 

      3   
(A)  ,  (B) (0, ) (C)  ,  (D)  0, 
 2 2  4 4   2

Q.42 If the variable line y = kx + 2h is tangent to an ellipse 2x2 + 3y2 = 6, then locus of P(h, k) is a
conic C whose eccentricity equals
5 7 7 7
(A) (B) (C) (D)
2 3 2 3

FINAL STEP Page # 6


MATHEMATICS
1
x5 5x 4 10 x3 10 x 2 5x 10 4 ln x 1  dx = ep  1 , then p + q equals
Q.43 If e 5eq
1
(A) 23 (B) 32 (C) 45 (D) 53

(tan x  4) sec 2 x
Q.44  tan x (tan 2 x  4) dx is
 tan x 
(A) tan 1  2
  ln (tan x )  ln 4  tan x  C
 2 
1  tan x 
(B) tan 1 2
  ln (tan x )  ln 4  tan x  C
2  2 
1  tan x 
(C) tan 1 2
  ln (tan x )  ln 4  tan x  C
2  2 

1  tan x  2
(D) tan    ln (tan x )  ln 4  tan x  C
 2 

k
Q.45 Let Pk be a point whose x-coordinate is 1 + (k = 1, 2, 3, ....., n) on the curve y = ln x. If coordinates
n

1 n
of the point A on the curve is (1, 0), then the value of Lim
n 
 APk  2 is
n k 1

1 1 1 1
(A) + 2 (ln 2 – 1)2 (B) – 2(ln 2 + 1)2 (C) + 3 (ln 2 – 1)2 (D) – 3 (ln 2 + 1)2
3 3 2 2

Q.46 The graphs of x2 + y2 = 4 + 12x + 6y and x2 + y2 = kx + 4x + 12y where k satisfies a  k  b and


for no other values of k. The value of (b – a) is
(A) 5 (B) 68 (C) 104 (D) 140

2
1  sin x
Q.47 The value of definite integral  1  sin x
dx , is
0

1
(A) 1 (B) ln 2 (C) ln 2 (D) ln 2
2


Q.48 If two opposite sides of tetrahedron of volume 12 are 3 and 4 and angle between them is ,
6
then the shortest distance between them is
(A) 6 (B) 12 (C) 18 (D) 24

2
( x  1) e 2 x 3
Q.49 The value of definite integral  1  ( x  1) (e2 x 3  1)
dx is equal to
1
1  1
(A) (B) (C) (D) 1
2 4 8
FINAL STEP Page # 7
MATHEMATICS
Q.50 L et C1 and C2 are circles defined by x2 + y2 – 20x + 64 = 0 and x2 + y2 + 30x + 144 = 0.
The length of the shortest line segment PQ that is tangent to C1 at P and to C2 at Q is
(A) 15 (B) 18 (C) 20 (D) 24

3 x 3 h 3x

t sin t dt   t 2 sin t dt
2

Q.51 If Lim x h x has the value equal to x2 (k sin 3x – sin x) then k is equal to
h 0 h
(A) 3 (B) 9 (C) 27 (D) none

Q.52 Let P1 : 2x – y + z – 2 = 0 and P2 : x + 2y – z = 3. Then the equation of plane passing through the point
(– 1, 3, 2) and perpendicular to the planes P1 and P2 is
(A) x + 3y – 5z + 2 = 0 (B) x + 3y + 5z – 18 = 0
(C) x – 3y – 5z + 20 = 0 (D) x – 3y + 5z = 0

6
1  sin x
Q.53  cos x
dx equals
0

2 3   2 3  2 3   2 1 
(A) 2 2 ln   (B) 2 ln  
 (C) 2 ln   (D) 2 ln  
2  1  2  1 2  1  2  3 
       

Q.54 If all three planes


P1 : (a + 1)3 x + (a + 2)3 y + (a + 3)3 z = 0,
P2 : (a + 1) x + (a + 2) y + (a + 3) z = 0
P3 : x + y + z = 0
pass through a point other than origin, then a equals
(A) – 3 (B) 2 (C) – 2 (D) 3

Q.55 Consider two complex numbers, z1 =  3 + 2 i & z2 = 2  3 i , z is a complex number such that ,
 z  z1   1 
arg  z  z  = cos1   then , the locus of z is a circle whose radius is
 2   10 

5 5 5 2 3 5
(A) 3 7 (B) (C) (D)
3 3 2

FINAL STEP Page # 8


MATHEMATICS
[COMPREHENSION TYPE]
Paragraph for question nos. 56 & 57

From a lot containing 3 defective and 6 good items, 4 items are chosen at random which are to be checked
for their quality.As a result of this inspection, first 2 of the 4 items chosen found to be both good.

Q.56 The probabilitythat the 4 chosen items for inspection where all good, is
1 2 3 4
(A) (B) (C) (D)
7 7 7 7

Q.57 If the checking for quantity continues then the probability that the third inspected item will also be good,
is
1 2 3 4
(A) (B) (C) (D)
7 7 7 7

Paragraph for Question Nos. 58 to 60

Consider a plane mirror P1 having equation as z = 0. A ray of light emerging from the point
x 1 y  2 z  3
P(1, 2, 3) and travelled along the line   gets reflected from the plane mirror P1 and
3 2 1
reflected line is denoted by L2 .

Q.58 The equation of the line L2, is


 
(A) r  ( 8iˆ  4ˆj)   (3iˆ  2ˆj  k)
ˆ (B) r  ( 8iˆ  4ˆj)   (3iˆ  2ˆj  k)
ˆ
 
(C) r  ( 4iˆ  8j)
ˆ   (3iˆ  2ˆj  k)
ˆ (D) r  ( 8iˆ  4ˆj)   (3iˆ  2ˆj  k)
ˆ

Q.59 The equation of the plane ‘P2’ containing both the lines L1 and L2, is
(A) 2x – 3y + 4 = 0 (B) 3x – 2y + 4 = 0
(C) 2x + 3y + 4 = 0 (D) 3x + 2y + 4 = 0

Q.60 The equation of the plane P3 passing through the intersection of planes P1 and P2 and bisecting the angle
between P1 and P2, is
(A) 2x – 3y  13 z + 4 = 0 (B) 3x – 2y  11 z + 4 = 0
(C) 2x – 3y  11 z + 4 = 0 (D) 2x + 3y  13 z = 4 = 0

Paragraph for Question Nos. 61 to 63

Consider a twice differentiable function f(x) of degree four symmetrical to line x = 1 defined as
f : R  R and f " (2) = 0.

Q.61 The sum of roots of the cubic, f '(x) = 0, is


(A) 0 (B) 1 (C) 2 (D) 3

Q.62 If f(1) = 0, f(2) = 1, then the value of f(3), is


6 7 8 9
(A) (B) (C) (D)
5 5 5 5

FINAL STEP Page # 9


MATHEMATICS
Q.63 The distance between two horizontal tangents of the curve f(x), is
6 7 8 9
(A) (B) (C) (D)
5 5 5 5

Paragraph for Question Nos. 64 & 65


 x2 ; 0  x  1/ 2

Let us define a function g(x) =  2 1 and another function f(x) such that the Area
(x  1) ;  x 1
 2
enclosed by curves g(x) and x-axis between the ordinates x =  and x =  + f() is half the area enclosed
by g(x) and x-axis.
1 1
(where 0    and    f () < 1)
2 2

 f  x   x  1
3

Q.64 The value of Lt , is


1  1
x  
x
2
 2

3 1 1
(A) (B) (C) (D) 1
4 2 4

Q.65 Let at x = , f(x) has a local extremum, then the value of 3, is
(A) 16 (B) 8 (C) 4 (D) 1

Paragraph for question nos. 66 to 68

Consider z = x + iy where x, y  R and i =  1 . Let A (z 0 ) be a complex number


satisfying | z – 2i | = Im(z) and arg(z0 + 1) = .
[Note: Im(z) and arg(z) denote imaginary part of z and argument of z respectively.]

Q.66 The locus of P(z) represents


(A) a parabola whose length of latus-rectum equals 2.
(B) a parabola whose vertex is (1, 0).
(C) a parabola whose equation of directrix is x = 0.
(D) a parabola whose focus is (0, 2).

Q.67 If is minimum, then


2i  z 0 z  2i  2i  z 0 
(A)  0 =0 (B) I m   = 0
1  2i 2i  1  1  2i 
 2i  z 0  2i  z 0 z0  2i
(B) Re   = 0 (D)  =0
 1  2i  1  2i 2i  1

FINAL STEP Page # 10


MATHEMATICS

 z  2i  | z 0  2i |
Q.68 If  is maximum, then the circle z   0  = contains points whose
 2  2
(A) Im(z)  0 (B) Re(z)  1 (C) Re(z)  0 (D) Im(z)  1
[where Re(z) denotes real part of z.]

Paragraph for question nos. 69 & 70

Let A = {1, 3, 5, 7} and B = {2, 4, 6, 8} and f : A  B be a function defined from set A to set B.

Q.69 The number of possible functions defined from setAto set B such that
i + f (i) < 10  i = {1, 2, 3, 4}, is
(A) 192 (B) 144 (C) 216 (D) 256

Q.70 The number of possible functions defined from set Ato set B such that
f (i)  i + 1  i = {1, 2, 3, 4}, is
(A) 192 (B) 144 (C) 216 (D) 256

Paragraph for question nos. 71 & 72

Let C1 : x2 + y2 – 2x + 2y + 1 = 0 and C2 : x2 + y2 – 2x + 2y = 0 are two given circles. From a moving


point P on C2, tangents are drawn to C1 at A and B.

Q.71 The locus of orthocentre of PAB is


(A) x2 + y2 – 2x + 2y + 1 = 0 (B) x2 + y2 – 2x + 2y = 0
(C) x2 + y2 + 2x – 2y = 0 (D) x2 + y2 – 2x – 2y + 1 = 0

Q.72 The locus of circumcentre of PAB is


(A) 2x2 + 2y2 – 3x +3y = 0 (B) x2 + y2 – 2x + 2y – 3 = 0
(C) x2 + y2 – 3x + 3y + 1 = 0 (D) 2x2 + 2y2 – 4x + 4y + 3 = 0

Paragraph for Question no. 73 & 74

Let z1 and z2 are two complex numbers satisfying the equation 3z12 – 2z1z2 + 2z22 = 0 and
z 2
also Re 1   0 . Also. P1, P2 and O are points in complex plane corresponding to z1, z2 and
z
 1  2 
origin respectively.

Q.73 The value of z1 is equal to

(A) 1 (B) 5 (C) 2 (D) 2 2

Q.74 Area of triangle OP1P2 is


(A) 5 (B) 2 5 (C) 3 5 (D) 4 5

FINAL STEP Page # 11


MATHEMATICS
Paragraph for Question no. 75 to 77

x2 y2 x2 y2
Consider the parabola y2 = 20x, ellipse   1 and hyperbola   1
16 9 29 4

Q.75 The equation of common tangents to all of them, is


(A) y = ± (x – 5) (B) y = ± (x + 3) (C) y = ± (x + 5) (D) y = ± (x + 4)

Q.76 Area of the quadrilateral formed by common tangents and chord of contacts of ellipse and parabola
respectively, is
2149 2191 2194 2419
(A) (B) (C) (D)
25 25 25 25
x2 y2
Q.77 Equation of concentric ellipse touching auxiliary circles of the ellipse   1 and hyperbola
16 9
x2 y2
  1 , with major axis as the x-axis, is
29 4
x 2 y2 x 2 y2 x 2 y2 x 2 y2
(A)  1 (B)  1 (C)  1 (D)  1
16 9 29 16 25 16 16 4

Paragraph for question nos. 78 to 81

Let f : R  R be a function defined as f(x) = (x – 1) (x + 2) (x – 3) (x – 6) – 100 . If g(x) is a polynomial


g( x )
of degree  3 such that  f (x ) dx does not contain any logarithmic function and g(– 2) = – 10.
Q.78 The equation f(x) = 0 has
(A) all four distinct real roots. (B) three distinct real roots.
(C) two real and two imaginary roots. (D) all four imaginary roots.

Q.79 The function ' f ' is


(A) one-one onto (B) one-one into (C) many one onto (D) many one into

Q.80 The minimum value of f(x) is


(A) – 136 (B) – 100 (C) – 84 (D) – 68

4
g( x )
Q.81  f ( x) dx equals
0

 
(A) (B) (C)  (D) 2
4 2

FINAL STEP Page # 12


MATHEMATICS
Paragraph for Question no. 82 to 84

 5x 
 1   x2 1 0 0
 x x 0 0 
 
3
For x > 0, let A =  0 x 0  and B =  0 0  be two matrices.
   x 
 0 0 16 
 1
   0 0
 4 

Three other matrices X, Y and Z are defined as


X = (AB)–1 + (AB)–2 + (AB)–3 + ................ (AB)–n, Y = Lim X and Z = Y–1 – 2I,
n
where I is identity matrix of order 3.
[Note: tr (P) denotes the trace of matrix P.]

Q.82  
The value of det . adj ( 5 Y 1 ) is equal to
(A) (5!)2 (B) 53 (5!)2 (C) 5 (5!)2 (D) 52 (5!)2

Q.83 Least positive integral value of tr (AY) is equal to


(A) 8 (B) 7 (C) 6 (D) 5

Q.84 If tr (Z + Z2 + Z3 + ....... + Z10) = 2a + b where a, b  N, then least value of (a + b) is equal to


(A) 11 (B) 12 (C) 18 (D) 19

Paragraph for question No. 85 to 87

The system of equations


x + y + z = 5, x + 2y + 3z = 9, x + 3y + z = 

Q.85 Has a unique solution if


(A)  = 5,  = 13 (B)  5 (C)  = 5,  13 (D)  13

Q.86 Has no solution if


(A) = 5,  = 13 (B) = 5 (C)  = 5,  13 (D)  = 13

Q.87 Has infinite number of solution if


(A)  = 5,  = 13 (B) 5 (C) = 5,  13 (D) none of these

Paragraph for Question no. 88 to 90

Let P(x) = x5 – 9x4 + px3 – 27x2 + qx + r (p, q, r  R) be divisible by x2 and ,  and 


P( x )
are the positive roots of the equation = 0.
x2

Q.88 The value of (p + q + r) is equal to


(A) 9 (B) 27 (C) 81 (D) 108

FINAL STEP Page # 13


MATHEMATICS
Q.89 If  – 1,  + 3 and  + 7 are the first three terms of a sequence whose sum of first n terms

1
is given by Sn then  Sn · Sn 1
is equal to
n 2

1 1
(A) 1 (B) (C) (D) 2
4 2
 1 1 1 1 
Q.90 The value of Lim   2 2 2
 3 3 3
 ........  n n
 is equal to
n 
n  p  q  r p  q  r p  q  r p  q  r
 
1 1 25 26
(A) (B) (C) (D)
26 27 26 27

[REASONING TYPE]
Q.91 Statement-1: If f(x) = x2 + 10 sin x, then there exists some c  R such that f(c) = 1000.
Statement-2: If g(x) is defined on [a, b] such that g(a) > 0 and g(b) < 0, then there exists some
x0  (a, b) for which g(x0) is zero.
(A) Statement-1 is true, statement-2 is true and statement-2 is correct explanation for statement-1.
(B) Statement-1 is true, statement-2 is true and statement-2 is NOT the correct explanation for statement-1.
(C) Statement-1 is true, statement-2 is false.
(D) Statement-1 is false, statement-2 is true.

Q.92 Let A(x1, y1), B(x2, y2) and C(x3, y3) are the vertices of a triangle ABC.
Statement-1: If angle C is obtuse then the quantity (x3 – x1)(x3 – x2) + (y3 – y1)(y3 – y2) is negative.
Statement-2: Diameter of a circle subtends obtuse angle at any point lying inside the semicircle.
(A) Statement-1 is true, statement-2 is true and statement-2 is correct explanation for statement-1.
(B) Statement-1 is true, statement-2 is true and statement-2 is NOT the correct explanation for statement-1.
(C) Statement-1 is true, statement-2 is false.
(D) Statement-1 is false, statement-2 is true.

Q.93 Statement-1: If (x1 – x2)2 + (y1 – y2)2 = a2


(x2 – x3)2 + (y2 – y3)2 = b2
and (x3 – x1)2 + (y3 – y1)2 = c2
2
x1 y1 1
then x2 y 2 1 = (a + b + c)(b + c – a)(c + a – b)(a + b – c).
x3 y3 1

x y1 1
1 1
Statement-2: Area of triangle ABC, where A(x1, y1), B(x2, y2), C(x3, y3), is x2 y2 1 .
2 x y3 1
3

(A) Statement-1 is true, statement-2 is true and statement-2 is correct explanation for statement-1.
(B) Statement-1 is true, statement-2 is true and statement-2 is NOT the correct explanation for statement-1.
(C) Statement-1 is true, statement-2 is false.
(D) Statement-1 is false, statement-2 is true.

FINAL STEP Page # 14


MATHEMATICS
Q.94 Statement-1: Locus of the image of a point A(0, 0) in the line (2a + b)x + (3a – 2b)y + (b – 5a) = 0,
is x2 + y2 + 2x – 2y = 0, (where a and b are parameters).
Statement-2: If the line L is the angle bisector of the lines l1 and l2 then image of anypoint on l1 in the
line L lies on l2.
(A) Statement-1 is true, statement-2 is true and statement-2 is correct explanation for statement-1.
(B) Statement-1 is true, statement-2 is true and statement-2 is NOT the correct explanation for statement-1.
(C) Statement-1 is true, statement-2 is false.
(D) Statement-1 is false, statement-2 is true.

Q.95 Statement-1: Let f : R  R be a continuous function such that Lim f ( x ) = 10, then f (x) must
x

be many-one function.
Statement-2: If any function g(x) is continuous in its domain and Lim g ( x ) = c (where c is a finite
x

number) then g(x) must be many-one function.


(A) Statement-1 is true, statement-2 is true and statement-2 is correct explanation for statement-1.
(B) Statement-1 is true, statement-2 is true and statement-2 is NOT the correct explanation for statement-1.
(C) Statement-1 is true, statement-2 is false.
(D) Statement-1 is false, statement-2 is true.

 1   1 
Q.96 Statement-1 : sin 1    tan 1  
 e  

Statement-2 : sin–1 x > tan–1 y for x > y, x, y  (0, 1).


(A) Statement-1 is true, statement-2 is true and statement-2 is correct explanation for statement-1.
(B) Statement-1 is true, statement-2 is true and statement-2 is NOT the correct explanation for statement-1.
(C) Statement-1 is true, statement-2 is false.
(D) Statement-1 is false, statement-2 is true.

Q.97 Statement-1: The set of real values of y for which real x satisfy
yx2 + y2 x+ 2 = 0 is (–0]  [2, ).
Statement-2: If x is real then the discriminant of quadratic equation px2 + qx + r = 0 is non-negative,
where p, q, r  R and p  0.
(A) Statement-1 is true, statement-2 is true and statement-2 is correct explanation for statement-1.
(B) Statement-1 is true, statement-2 is true and statement-2 is NOT the correct explanation for statement-1.
(C) Statement-1 is true, statement-2 is false.
(D) Statement-1 is false, statement-2 is true.

Q.98 Statement-1: Let A be 2 × 3 matrix and B is 3 × 2 matrix such that det. (AB) = 10, then the value
of det. (BA), is equal to zero.
Statement-2: If A and B are two square matrices of order 3, then det. (AB) = det.(A) · det.(B).
(A) Statement-1 is true, statement-2 is true and statement-2 is correct explanation for statement-1.
(B) Statement-1 is true, statement-2 is true and statement-2 is NOT the correct explanation for statement-1.
(C) Statement-1 is true, statement-2 is false.
(D) Statement-1 is false, statement-2 is true.

FINAL STEP Page # 15


MATHEMATICS
Q.99 Statement-1: If tangents drawn to the parabola y2 = 4x at the points whose abscissa are in the
ratio 16 : 1, then the locus of the point of intersection of these tangents is a parabola P1
whose directrix lies on positive x-axis. Then the area enclosed by the parabola
P1 and x2 = 12y is equal to 25.
16ab
Statement-2: Area enclosed by the parabolas y2 = 4ax (a > 0) and x2 = 4by (b > 0) is equal to .
3
(A) Statement-1 is true, statement-2 is true and statement-2 is correct explanation for statement-1.
(B) Statement-1 is true, statement-2 is true and statement-2 is NOT the correct explanation for statement-1.
(C) Statement-1 is true, statement-2 is false.
(D) Statement-1 is false, statement-2 is true.

 
Q.100 Statement-1: The value of Lim ( x  2) tan 1 ( x  2)  ( x tan 1 x ) is equal to .
x 
Statement-2: If f(x) is derivable in [a, b] then  atleast one c  (a, b) such that
f ( b)  f (a )
f '(c) = .
ba
(A) Statement-1 is true, statement-2 is true and statement-2 is correct explanation for statement-1.
(B) Statement-1 is true, statement-2 is true and statement-2 is NOT the correct explanation for statement-1.
(C) Statement-1 is true, statement-2 is false.
(D) Statement-1 is false, statement-2 is true

Q.101 Statement 1: Consider two curves


C1 : zz  iz  iz  b  0
and C2 : zz  (1  i) z  (1  i) z  4  0 where (b R, z = x + iy and i =  1 ).
If C1 and C2 intersects orthogonally then b = –2.
Statement 2: If two curves intersects orthogonally then the angle between the tangents at all their

points of intersection is .
2
(A) Statement-1 is true, statement-2 is true and statement-2 is correct explanation for statement-1.
(B) Statement-1 is true, statement-2 is true and statement-2 is NOT the correct explanation for statement-1.
(C) Statement-1 is true, statement-2 is false.
(D) Statement-1 is false, statement-2 is true

 
Q.102 Statement 1: The circles x2 + y2 = 9 and 2 x  3 x  1  2 y y  6  0 touches each other
internally.
Statement 2: Circle described on the focal distance as diameter of the ellipse 8x2 + 9y2 = 72
touches the auxiliary circle x2 + y2 = 9 internally.
(A) Statement-1 is true, statement-2 is true and statement-2 is correct explanation for statement-1.
(B) Statement-1 is true, statement-2 is true and statement-2 is NOT the correct explanation for statement-1.
(C) Statement-1 is true, statement-2 is false.
(D) Statement-1 is false, statement-2 is true

FINAL STEP Page # 16


MATHEMATICS

 
Q.103 Statement-1: The incentre of triangle formed by the lines L1: x cos    y sin   – 2 = 0 ,
 10   10 

   


L2 : x cos    y sin   + 2 = 0 and L3 : x cos    y sin   + 2 = 0 is (0, 0).
8 8 5 5
Statement-2: The incentre I of a triangleABC is the point of concurrency of 3 internal angle bisectors
of triangleABC.
(A) Statement-1 is true, statement-2 is true and statement-2 is correct explanation for statement-1.
(B) Statement-1 is true, statement-2 is true and statement-2 is NOT the correct explanation for statement-1.
(C) Statement-1 is true, statement-2 is false.
(D) Statement-1 is false, statement-2 is true.

2
Q.104 Statement-1: If the function f(x) = x  a x  b is derivable for all x  R, then number of
ordered pairs (a, b) is equal to 1.
Statement-2: If f(x) is a polynomial function then f ( x ) is non-derivable at the points where f(x)
vanishes.
(A) Statement-1 is true, statement-2 is true and statement-2 is correct explanation for statement-1.
(B) Statement-1 is true, statement-2 is true and statement-2 is NOT the correct explanation for statement-1.
(C) Statement-1 is true, statement-2 is false.
(D) Statement-1 is false, statement-2 is true.

Q.105 Statement-1: Numerically greatest coefficient in the expansion of (1 + 5x)8 is 8C4 · 54.
Statement-2: Greatest coefficient in the expansion of (1 + x)2n is the middle term.
(A) Statement-1 is true, statement-2 is true and statement-2 is correct explanation for statement-1.
(B) Statement-1 is true, statement-2 is true and statement-2 is NOT the correct explanation for statement-1.
(C) Statement-1 is true, statement-2 is false.
(D) Statement-1 is false, statement-2 is true.

Q.106 Statement-1: The value of


 20  20   20 19   20 18   20 17   20 10 
 0  10    1  9    2  8    3  7   ..........   10  0  = 0
              

n n
where  r  = C r .
 
n
Statement-2: C 0  n C1  n C 2  n C3  .........  (1) n n C n = 0.
(A) Statement-1 is true, statement-2 is true and statement-2 is correct explanation for statement-1.
(B) Statement-1 is true, statement-2 is true and statement-2 is NOT the correct explanation for statement-1.
(C) Statement-1 is true, statement-2 is false.
(D) Statement-1 is false, statement-2 is true.

FINAL STEP Page # 17


MATHEMATICS
Q.107 Let Abe a 2 × 2 matrix with real entries. Let I be the 2 × 2 identity matrix. Denote Tr (A) as the sum of
diagonal entries of A. Assume that A2 = I.
Statement 1: If A  I and A  –I, then det A = –1.
Statement 2: If A  I and A  –I, then Tr (A)  0.
(A) Statement-1 is true, statement-2 is true and statement-2 is correct explanation for statement-1.
(B) Statement-1 is true, statement-2 is true and statement-2 is NOT the correct explanation for statement-1.
(C) Statement-1 is true, statement-2 is false.
(D) Statement-1 is false, statement-2 is true.

Q.108 Consider the function f(x) = x  1  x  4 for all x  R.


Statement-1: f '() = 0.

Statement-2: f(c) = , for some c  R.
2
(A) Statement-1 is true, statement-2 is true and statement-2 is correct explanation for statement-1.
(B) Statement-1 is true, statement-2 is true and statement-2 is NOT the correct explanation for statement-1.
(C) Statement-1 is true, statement-2 is false.
(D) Statement-1 is false, statement-2 is true.

Q.109 Let C be a circle with centre 'O' and HK is the chord of contact of pair of the tangents from point A. OA
intersects the circle C at P and Q and B is the midpoint of HK, then
Statement-1 : AB is the harmonic mean ofAP and AQ.
because
Statement-2 :AK is the Geometric mean ofAB andAO and OAis the arithmetic mean ofAP andAQ.
(A) Statement-1 is true, statement-2 is true and statement-2 is correct explanation for statement-1.
(B) Statement-1 is true, statement-2 is true and statement-2 is NOT the correct explanation for statement-1.
(C) Statement-1 is true, statement-2 is false.
(D) Statement-1 is false, statement-2 is true.

Q.110 Statement-1: The number of ways in which we can choose two distinct integers from 1 to 100 such
that the difference between them is at most 10 is (100C2 – 90C2).
Statement-2: Number of non-negative integral solution of the equation x1 + x2 + ..... + xr = n,
xi  0,  i = 1, 2, ......, n is equal to n + r – 1Cr – 1.
(A) Statement-1 is true, statement-2 is true and statement-2 is correct explanation for statement-1.
(B) Statement-1 is true, statement-2 is true and statement-2 is NOT the correct explanation for statement-1.
(C) Statement-1 is true, statement-2 is false.
(D) Statement-1 is false, statement-2 is true.

[MULTIPLE OBJECTIVE TYPE]


Q.111 Let f : (0, )  (0, ) be a derivable function and F (x) is the premitive of f (x) such that
2F( x )  f ( x )  = f 2 (x) for any real positive x.
f (x)
(A) f is strictly increasing (B) Lim =1
x  x
(C) f is strictly decreasing (D) f is non monotonic

Q.112 The area of a triangle is 5. Two of its vertices are (2, 1) & (3, 2). The third vertex lies on y = x + 3.
The third vertex, is

(A)  , 
7 13
(D)   , 
3 3
(B) (– 2, 1) (C) (2, 5)
2 2   2 2
FINAL STEP Page # 18
MATHEMATICS
Q.113 Which of the following statement(s) is(are) correct?
    
(A) If v  w = 0 for all vectors w , then v = 0 .
    
(B) If a  b  3 c is a null vector where a and b are unit vectors such that angle between
   
a and b is , then the magnitude of c is 1.
3
     
(C) If ( u  v) · ( u  v) = 0, then u  v .
   
(D) If a and b are two unit vectors then the maximum value of 2a  b is equal to 3.

x3 y6 z x2 y 1 z6


Q.114 Let L1 :   and L2 :   be two lines in R3.
4 3 2  4 1 1
Which of the following statement(s) is (are) correct?
(A) L1, L2 are coplanar .
(B) L1, L2 are skew lines.
(C) Shortest distance between L1 and L2 is 9.
(D) ( î  4ˆj  8k̂ ) is a vector perpendicular to both L and L .
1 2

Q.115 If  is the angle between the pair of tangents drawn from (c, 0) (where c > 0) to the circle x2 + y2 = 1
then which of the following conclusion(s) is/are true?
 5   3 
(A) if    ,    c  1, 6  2
 6 
  
(B) if    ,    c  1, 5  1
 5 

   
(C) if    ,    c  1,
2 
 2  (D) if    ,    c  (1, 2)
3 

Q.116 Which of the following statement(s) is(are) correct?


(A) If f (x) is differentiable  x  [0, 1], then f ' (x) must be bounded in [0, 1].
(B) There exist a bijective function f : [0, 1]  [0, 1] which is not continuous.
(C) Let f : [– 1, 2]  R be defined as f(x) = x2 – cos x + 4, then f(c) = 2 for some c  (– 1, 2).
1
(D) If f (x) = 3 + x + ex, then (f – 1)' (4) is equal to .
2

Q.117 Which of the following statements is/are incorrect?


(A) Two circles always have a unique common normal.
(B) Radical axis is always perpendicular bisector to the line joining the centres of two circles.
(C) Radical axis is nearer to the centre of circle of smaller radius.
(D) Two circles always have a radical axis.

Q.118 Three distinct lines are drawn in a plane. Suppose there exist exactly n circles in the plane tangent to all
the three lines, then the possible values of n is/are
(A) 0 (B) 1 (C) 2 (D) 4

Q.119 Let f (x) be twice differentiable function such that f '' (x) < 0 in [0, 2]. Then
(A) f (0) + f (2) = 2 f (c), 0 < c < 2 (B) f (0) + f (2) = 2 f (1)
(C) f (0) + f (2) > 2 f (1) (D) f (0) + f (2) < 2 f (1)

FINAL STEP Page # 19


MATHEMATICS
Q.120 An ellipse is orthogonal to the hyperbola x2 – y2 = 2. The eccentricity of the ellipse is reciprocal of that
of the hyperbola. Then
(A) equation of the ellipse is x2 + 2y2 = 8.

(B) focus of the ellipse is at  4 2 , 0 
(C) equation of directrix of ellipse is x  4 2  0
(D) equation of director circle of ellipse is x 2 + y2 = 12.

Q.121 If the tangent at a point P1 (other than (0,0)) on the curve ax3 – y + b = 0 meets the curve again at P2.
The tangent at P2 meets the curve at P3 and so on . If the abscissae of P1, P2, P3 .....Pn form a G.P. then
(a, b) may be
(A) (1, 0) (B) (2 , 7) (C) (3, 5) (D) (4, 9)

Q.122 If three planes P1  2x + y + z – 1 = 0, P2  x – y + z – 2 = 0 and P3 x – y + 3z – 5 = 0 intersects


each other at point P on XOY plane and at point Q on YOZ plane, where O is the origin then
identify the correct statement(s)?
(A) The value of  is 4.
x 1 y 1 z
(B) Straight line perpendicular to plane P3 and passing through P is   .
4 1 3
(C) The length of projection of PQ on x-axis is 1.
 1 1 1 
(D) Centroid of the triangle OPQ is  , , 
3 2 2

 p 1 q
 x sin  x tan x , x  0 p, q  N
Q.123 Let f(x) =  x
0, if x0
If f(x) is derivable at x = 0 then the ordered pair (p, q) is
(A) (1, 2) (B) (2, 1) (C) (3, 2) (D) (2, 2)

Q.124 If | z – 2 | = min{| z |, | z – 4 | } then possible value(s) of Re(z) can be


(A) 1 (B) 2 (C) 3 (D) 4

 
n
Q.125 If 9  80 = 1 + f where I , n are integers and 0 < f < 1, then
(A) I is an odd integer (B) I is an even integer
 
n
(C) (I + f) (1  f) = 1 (D) 1  f = 9  80

Q.126 Which of the following statement is(are) true?


x 2 y2
(A) From any point on the directrix of an ellipse   1 (a > b), a pair of tangents are drawn
a 2 b2
to the auxiliary circle of the ellipse. The chord of contact will pass through the corresponding
focus of the ellipse.
(B) If two tangents to a parabola y2 = 4ax intersect on the line x = a then their chord of contact
always passes through the foot of directrix.

(C) If P(x, y) is such that it moves on a hyperbola (x  3)2  ( y  4)2  x 2  y2 = k2 + 1,


then number of possible integral values of k is equal to 3.
(D) A circle passing through 3 co-normal points on the parabola y2 = 4x also passes through (1, 0).

FINAL STEP Page # 20


MATHEMATICS
Q.127 A function y = f (x) satisfies the following conditions
(i) f (1) = 1
(ii) The tangent at any point of the graph of f (x) cuts off on the y-axis an intercept equal to the
square of the abscissa of the point of tangency.
Which of the following are correct?
(A) The graph of y = f (x) passes through the origin.
(B) y = f (x) denotes a conic where coordinates of focus are (1, 3/4)
(C) The area enclosed by f (x), and the line y + 3 = 0 is 32/3.
(D) y = f (x) has no maxima.


 
  
Q.128 The line of intersection of the planes r  3î  ˆj  k̂ = 1 and r  î  4ˆj  2k̂ = 2 is parallel to the vector
(A) 2î  7ˆj  13 k̂ (B)  2î  7ˆj  13 k̂ (C)  2î  7ˆj  13 k̂ (D) 2î  7ˆj  13 k̂

Q.129 The roots of equation x5 – 40x4 + x3 + x2 + x +  = 0 are real and in G.P. If the sum of their
reciprocals is 10, then  can be
1 1
(A) – 32 (B) (C) (D) 32
32 32
Q.130 In the expansion of (2x – 3y + z)10 , then which of the following statement(s) is/are correct?
(A) The expansion will contain 66 terms.
(B) Sum of all the coefficients is 0.
  10 !  9 
(C) The coefficient of x2y3z5 equals   .
 5! 
(D) Sixth term is the middle term of the expansions.

Q.131 Let f(x) = (x2 + 2x + 3)2 + 2(x2 + 2x + 3) + 3, then which of the following statement(s) is (are) correct?
(A) The equation f(x) = 0 has no real roots.
(B) The equation f(x) = 0 has two real roots and two imaginary roots.
(C) The minimum value of f(x) equals 11.
(D) The minimum value of f(x) equals 12.
Q.132 Consider the equation in real number x and a real parameter p, as 1  x  x  2  x  4 = p.
If the equation to have exactly two solutions, the parameter p can be
(A) 1 (B) 3 (C) 5 (D) 7
Q.133 Given that x1, x3 are roots of the equation ax2 – 4x + 1 = 0 and x2, x4 are roots of the equation
bx2 – 6x + 1 = 0. If x1, x2, x3, x4 are in harmonic progression, then
1 1
(A) 3a – b = 1 (B) a2 + b2 = 73 (C) 2a < 3b (D) 
a b
Q.134 If 100C6 + 4 100C7 + 6 100C8 + 4 100C
9 + 100C
10 has the value equal to xCy, then the value of
(x + y) can be
(A) 114 (B) 115 (C) 198 (D) 199

Q.135 If 1 + sin  + sin2 + .........  = 4  2 3 where 0 <  < ,   , then
2
  5 2
(A)  = (B)  = (C)  = (D)
6 3 6 3

FINAL STEP Page # 21


MATHEMATICS
10
 2 1 
Q.136 The value of the constant term in the trinomial  x  2  2  is also equal to
 x 
(A) number of different dissimiliar terms in (x1 + x2 + ......... + x10)10.
(B) (10C0)2 + (10C1)2 + (10C2)2 + ........ + (10C10)2
(C) coefficient of x10 in (1 – x)20.
(D) number of linear arrangements of 20 things of which 10 alike of one kind and rest all are different,
taken all at a times.
Q.137 Which of the following is(are) correct?
n 2 1
 2n  32  n 1  x x  1
(A) Lim  2  is equal to e . (B) Lim x      equal to .
n  2 n  n  1  x   e  x  1   2e
   
(C) The coefficient of x50 in (1 + x2)25 (1 + x25) (1 + x40) (1 + x45) (1 + x47) is equal to 1 + 25C5.
(D) IfA, B are non-singular and symmetric matrices such thatAB = BAthen A–1 B–1 is symmetric matrix.
Q.138 If the coefficients of (4r – 4)th term and (r + 14)th term in the expansion of (1 + x)28 are equal,
then the value of r, is(are)
(A) 4 (B) 5 (C) 6 (D) 7
Q.139 Let the complex number z = x + iy (where x, y  R) satisfy
cot–1( log3 | 2z + 1 | ) > cot–1 (log3 | 2z – 1 | ) , then x can be
(A) – 1 (B) – 2 (C) 1 (D) 2
100
Q.140 The coefficient of x50 in the expansion of  100 Ck (x  2)100k 3k is also equal to
k 0
(A) number of ways in which 50 identical books can be distributed in 100 students, if each student
can get atmost one book.
(B) number of ways in which 100 different white balls and 50 identical red balls can be arranged
in a circle, if no two red balls are together.
(C) number of dissimilar terms in (x1 + x2 + x3 + ....... + x50)51.
2 · 6 · 10 · 14 · ........ 198
(D)
50!

[MATCH THE COLUMN]


Q.141 Column-I Column-II
n n
(A) If the sum   rs 2r · 3s = 1554 then n is equal to (P) 1
r 1 s1

 0 if r  s
where rs =  (Q) 2
 1 if r  s

(B) If the equation | x  2 | 2 = r has 3 distinct solutions, (R) 3


(C) The sum of all real solution of the equation (S) 4

log2(– x2 + 7x – 10) + 3 cos   x 2  7   1 = 1.


 
then the value of r is

FINAL STEP Page # 22


MATHEMATICS
Q.142 Column-I Column-II
(A) 2 2 2 2
Let C1 : x + y = 676 and C2 : x + y = 10x be two circles. (P) 3
The difference between the lengths of the longest and shortest chords
of the larger circle that are tangent to the smaller circle, is (Q) 4
1
(B) The gradient of the linear function f(x) = 1 + x  f ( t 2 ) dt , is (R) 5
1

(C) Number of solutions of the equation sin6x + cos3x = 1 (S) 6


which lie in the interval [0, 2) is relatively prime with

Q.143 Column-I Column-II

3
x2
(A) If the definite integral e dx vanishes, then the number of values of , is (P) 7

(B) If (a, b) are the co-ordinates of the middle point of the chord cut off by (Q) 5
2x – 5y + 18 = 0 on the circle x2 + y2 – 6x + 2y – 54 = 0, then (a + b)
is twin prime with (R) 4
10
(C) In the equation x2 – x + c = 0, one root is the square of the other root.
9
If c > 0 is the rational number m n  in the simplest form, then the value (S) 3
of (m + n) is divisible by
(D) A possible values of 'a' for which f (x) = loga(4ax – x2) is monotonically (T) 2
3 
increasing for every x   , 2 , is
2 

Q.144 Column-I Column-II


3i
(A) Let z be a complex number which satisfies z2 = 1 – , (P) 1
4
p
and the value of | (Re z) (Im z) | is equal to
q
(where p and q are coprime), then the value of (q – p) is equal to
(B) A fair dice is rolled 3 times. If the chance that the last 2 rolls is atleast (Q) 5
p
as large as the roll before it is then p equals (R) 7
27
2
x
(C) Number of real solution(s) of the equation 2x  log 2 x = 2x + 1 , is (S) 9

FINAL STEP Page # 23


MATHEMATICS
Q.145 Column-I Column-II
(A) Let A, B, C, D, E, F be a convex regular hexagon and G be its center. (P) 0
If s denotes the number of straight lines that can be drawn with
these 7 points and t denotes the numer of triangles that can be formed
with these 7 points, then the value of (t – s) is equal to (Q) 3

x2 y4 z4 x y3 z2


(B) If the lines = = and = = (R) 9
1 p 1 1 1 q
are coplanar and perpendicular, then (p + q) can be

(C) If the vector v with magnitude 6 is along the internal bisector of (S) 17
 
the angle between a  7 î  4ˆj  4k̂ and b  2î  ˆj  2k̂ ,
 
then the value of v · a is equal to

Q.146 Column-I Column-II


(A) Length of side of an equilateral triangle with one vertex at (P) 18 3
origin and inscribed in the parabola y2 = 12x, is
(B) Length of chord through origin of the parabola (Q) 24 3
y2 = 12x and having slope 1, is
(C) Length of normal chord of the parabola y2 = 12x (R) 7 5
subtending a right angled at focus, is
(D) Length of normal chord of the parabola y2 = 12x (S) 12 2
subtending a right angle at the vertex, is (T) 15 5

Q.147 Column-I Column-II


(A) The circle x2 + y2 – 6x – 10y + c = 0 does not touch or intersect the (P) 12
coordinate axes and the point (1, 4) is inside the circle.
The number of integral values of C is (Q) 6
(B) The chords of contact of the pair of tangents to the circle x2 + y2 = 1
drawn from any point on the line 2x + y = 4 pass through the point (, ),
then the value of (3 + 2), is (R) 4
y
(C) If M and m are the maximum and minimum values of for pair of real
x
number (x, y) which satisfy the equation (x – 3)2 + (y – 3)2 = 6, (S) 3
then the value of (M + m), is equal to
(D) Sum of all the radii of the circles touching the coordinate axes and the line
3x + 4y = 12, is
(E) Sum of the abscissa and ordinate of the centre of the circle touching the line (T) 2
3x + y + 2 = 0 at the point (–1, 1) and passing through the point (3, 5) is

FINAL STEP Page # 24


MATHEMATICS
Q.148 Column-I Column-II

x 1 1
2
(A) Let f(x) = sin(2 x ) 2 x 1 . If y = g(x) is the image (P) 2
x3 3x 4 1

of y = f(x) in y-axis, then find the absolute value of f (1) g (1).


(B) If three equations (Q) 4
(a + 1)3 x + (a + 2)3 y = (a + 3)3
(a + 1) x + (a + 2) y = a + 3
x+y=1
are consistent then the value of | a |, is (R) 6
1 1
(1  3t  2t 2 ) t  (1  3t  2 t 2 ) t
(C) If the value of Lim (S) 8
t0 t
3
is equal to ke then k equals

35
6
(D) The value of definite integral 

3 sin x  4 cos x dx is divisible by (T) 10

Q.149 Column - I Column - II


(A) If f(x) = ax3 – 9x2 + 9x + 3 is strictly increasing on R, (P) 98
then he number of integral values of k, k [– 5, 100], is
(B) Number of integers in the range of
f(x) = cos3x – 6 cos2x + 11 cos x – 6, x  (– , ), is (Q) 85
(C) If tangent at point P(4m2, 8m3) to the curve x3 – y2 = 0

4
is also normal to the curve at point Q, then the value of , is (R) 81
m4
(D) Let (sin  – cos ) (tan  + cot ) = 2 and
if (sin  + cos ) (tan  – cot ) = a  b (where a, b  N), (S) 26
then the value of (a2 + b2), is equal to (T) 25

Q.150 Let the coordinates of the feet of the perpendicular from the vertices of a triangleABC on the opposite
sides are D(20, 25), E(8, 16) and F(8, 9).
Column-I Column-II
(A) If the orthocentre of the triangle ABC, is (p, q) then p + q equals (P) 45
(B) If the equation of the line BC, is y = – x + c, then the value of c, is (Q) 35
(C) If the centroid of the triangle ABC, is (a, b) then the sum (a + b), is (R) 25
(D) If the circumcentre of the triangleABC, is (l, m), then (l + m) equals (S) 15
(T) 14
FINAL STEP Page # 25
MATHEMATICS
[INTEGER TYPE]
Q.151 Find the sum of all real values of k for which the equation
x 2  (7  k 2 ) x  7k 2  ( x  3)( x  3k  2) = 0 has atleast one real solution.

Q.152 Points A, B and O (O is origin ) lie in the plane such that AOB is 60°. Circle S0 with radius r0 = 6
is tangent to both OA and OB. If for all i  1, circle Si with radius ri is constructed such that

ri < ri – 1 and Si is tangent to OA, OB and Si –1 then find the value of  ri1 .
r 1

13 a 3
Q.153 If sin (30° + arc tan x) = and 0 < x < 1, the value of x is , where a and b are positive integers
14 b
with no common factors. Find the value of (a + b).

Q.154 Let ABC be a triangle with incircle O and side lengths 5, 8 and 9. Consider the other tangent line to
O and parallel to BC which intersectAB at B1 andAC and C1. If ra is the inradius of triangleAB1C1 and
rb and rc are similarly defined, then find the value of 11 (ra + rb + rc)2.

  2n  2  2 n  2  
Q.155 If  tan 1 2
 =  tan 1 k then find the value of k2.
 2
n 1  2  n 1 

p
Q.156 A coin is tossed 9 times. If that the probability of at least 5 consecutive heads is (where p and q are
q
in their lowest form), then find the value of (p + q).

Q.157 Consider the system of equations ax + y = b, bx + y = a, ax + by = ab where a, b  {0, 1, 2, 3, 4}.


Find the number of ordered pairs (a, b) for which system is consistent.

 
2 2
2 2
Q.158 If range of the function f(x) =  cos 1    sin 1   sin 1  
x x x
x  6 x  8 is [a2, b2],
 2 2  2 12
then find the value of 2(a + b).

Q.159 Find the number of values of x  (– , 2) satisfying the equation


logsin 2 x (2)  log cos2 x (2)  2 logsin 2 x (2) · logcos2 x (2)  0 .

x
Q.160 Compute the value of E = cot2 – where 4 tan3 x – 3 tan2 x + 4 tan x – 3 = 0, x (0, ).
4 6 10

A 2A m
Q.161 In a triangle ABC, cos cos B  cos cos C = sin A. If the measure of the angle A + 3C =
3 3 n
where m, n  N, then find the smallest value of (m + n).

Q.162 If the system of equation 3x2 + 2x – 1 < 0 and (3a – 2) x – a2x + 2 < 0 posseses solution, then
find the least natural number a.

FINAL STEP Page # 26


MATHEMATICS
 4 a 1  2
Q.163 Let A =  1 b  and B =  are two matrices satisfying the relation
  1 4 
A3 + 3A2B + 3AB2 + B3 = (A + B)3. If (A + B) =  (A–1 + B–1) then find the value of .

  1 
Q.164 If the sum of all the roots of the equation sin   log 3    = 0 in (0, 2) can be expressed as lowest
  x 
rational p q , then find the vaue of (p + q).

Q.165 'A' rolls a fair die until a six appears for the first time. Independently 'B' rolls a fair die until a six appears
m
for the first time. Let (expressed as lowest rational) is the probability that the number of times 'A' rolls
n
his die is equal to or within one of the number of times 'B' rolls his die. Find (m + n).

Q.166 A computer solved several problems in succession. The time it took the computer to solve each successive
problem was the same number of times smaller than the time it took to solve the preceding problem.
How many problems were suggested to the computer if it spent 63.5 min to solve all the problems
except for the first, 127 min to solve all the problems except for the last one, and 31.5 min to solve all the
problems except for the first two?

Q.167 Find the number of solution of the equation tan(7 cos x) = cot (7 sin x) in (0, 2).

4 9 16 
Q.168 If x, y, z are positive real numbers and x + y + z = 1, then find the minimum value of    .
x y z 

Q.169 A fair die is tossed six times. Let P be the chance that there will be three consecutive 6's exactly once in
k
six tosses. If P = , k  N. Find k.
3888

Q.170 Find the value of K  R for which the quadratic equation x2 + (2K + 1)x + (K2 + 2) = 0
has two real roots such that one root is twice the other root.

100
 k  100 a (2100 )  b
Q.171 If    Ck 
 k 1 c
where a, b, c  N,
k0

then find the least value of (a + b + c).

 2 4 .
Q.172 Find the numerical value of cot2 + cot2 + cot2
7 7 7
4m  6
Q.173 Number of possible integral value(s) of m for the equation sin x – 3 cos x = can be valid for
4m
some x  [0, 2].

Q.174 The twentieth term of an arithmetic sequence is log1020 and the thirty second term is log1032. If exactly
one term of the arithmetic sequence is a rational number which in the lowest form is p q  , p, q  N,
then find the value of (p + q).
FINAL STEP Page # 27
MATHEMATICS
Q.175 A fair coin is repeated by tossed. If the probability that the first time heads is tossed, twice in a row,
p
is on the 9th and 10th toss, is . Then find the value of p.
512

Q.176 Find the number of permutations using all the eight letters of the word "D E E P M A L A" if the letter
D precedes both E's, the letter L appears between the two E's and the letter M appears after the
letter L.

Q.177 Let a, b, c be the sides of a triangle ABC and  be its area. If a = 2,  = 3 and
a cos C + 3 a sin C – b – c = 0. Find the numerical value of (b + c).

Q.178 A fair six sided die is tossed three times, and the sum of the numbers appearing on the first two tosses
1
equals the number on the third toss. If P =   denotes the probability that the face 2 occurs atleast
k
once then find the sum of digits in k.

Q.179 A teacher suggests four possible books for has six students to read. Each of six students selects one of
the four books. Find the number of ways in which these selections can be made if each of these books
by atleast one student.

sin x sin 3x sin 9 x


Q.180 Let P = tan(27x) – tan x and Q = + + . If P = Q, then find the value
cos 3x cos 9 x cos 27 x
of .

Q.181 If k  I then find the number of values of x in [0, 2] which satisfy the equation
 k  2  2k  3  2
  sin x   cosec x  2 .
 2k  3   k 

Q.182 Find the number of integral values of b for which the equation
(3 sin x – 4 cos x)2 – (b2 + b + 5) | 3 sin x – 4 cos x | + (b3 + 3b2 + 2b + 6) = 0 has a real solution.

Q.183 If the true set of values of k such that the equation (tan–1x)2 + k(tan–1x) –  cot–1x = 0
 a b 
has no real solution is  ,  , then find the value of a2 + b2.
2 2

2
Q.184 Find the absolute value of (tan A tan 2A) + (tan 2A tan 4A) + (tan 4A tan A), where A = .
7

x2
Q.185 Let f (x) be a differential function  x  R satisfying f (x) = f ' (1) ex – 1 – f (0)x + . Find the global
2
minimum value of f (x).

Q.186 Let C be a curve which has the property that length of the perpendicular from origin upon any of its
tangent is equal to the abscissa of the point of tangency. If the curves passes through (2, – 2) and
the area enclosed by the curve above the x-axis is k, then find the value of k.

FINAL STEP Page # 28


MATHEMATICS

6
   b
Q.187 If the value of definite integral  ln 1  tan x tan 6  dx is ln   (where a, b, c  N and b & c are
a c
0
relatively prime), then find the value of (a + b + c).

Q.188 If 9 + f ''(x) + f '(x) = x2 + f 2(x), where f(x) is twice differentiable function such that
f ''(x) 0 x R and let P be the point of maxima of f(x) then find the number of tangents
which can be drawn from P to the circle x2 + y2 = 9.

1 1
Q.189 If f : [0, 1]  R is a continuous function satisfying
1
 
2 2
0 f (x ) dx  3  0 f ( x ) dx ,
1
then find the reciprocal of f   .
4

Q.190 Let f(x) be a function such that f '(x) = x3 and the line x + y = 0 is tangent to the graph of f(x).

Find the value of 4 · f ( x )  x 4 . 
3 2 5
Q.191 Find the local maximum value of the function f(x) = (x2 – 2x) ln x – x  4x  .
2 2

 
Q.192 Let g(x) be a real valued function defined on the interval  ,  such that
 2 2
sin x
et  
g(x) = e2x +  2
cos x  2 t sin x  t 2
dt  x   ,  . Also f(x) be the inverse function of g(x),
 2 2
0

 1
where 0  x  . Find the value of
2 f ' (1)2 + g(0) + g'(0) + g"(0).

Q.193 If the equation of plane containing the line x – y + z = 0 and x + y + z – 2 = 0 and which is farthest
from origin is ax + by + cz = d, where a, b, c, d  N, then find the minimum value of (a + b + c + d).

1 100 3
Q.194 If 1 1
 [sec x ] dx   [sec x ] dx = p –  sec (i) , then find the value of 'p'.
100 1 i 1

[Note: [k] denote the greatest integer function of k.]

Q.195 Let C be a circle that intersects each of the circles (x + 2)2 + y2 = 4, (x – 4)2 + (y – 2)2 = 4 and
(x – 4)2 + (y + 2)2 = 4 in exactlyone point and does not contain any of these circles inside it. If the radius
r of C is of the form of where p and q are prime find (p + q).

Q.196 Let slope of sides AB and BC of a ABC are roots of the equation x 2 – 4x + 1 = 0,
(ABC lying in the first quadrant and B is origin) and inradius of the triangle is 1 unit.
If co-ordinates of its incentre are (a, b) then find (a2 + b4).

FINAL STEP Page # 29


MATHEMATICS
1 1
xn xn
Q.197 Let In =  x 2012  1 dx and Jn =  x 2013  1 dx  n > 2012, n  N if the matrix A = [aij] 3 × 3, where
0 0

I I , i j J 2016  j  J j  3 , i  j
aij =  2012i i and the matrix B = [bij]3 × 3 , where bij = 
i j .
 0, i j 0,
Find the value of trace(A–1) + det. (B–1).

1  cos 1  cos (1  cos x ) 


Q.198 If Lim exists and has a non-zero value, find the value of 'a'.
x 0 xa

Q.199 Let the set A consists of 5 and set B consists of 4 distinct elements. If m denotes the number of NOT
ONTO functions defined from Ato B and n denotes the number of functions from Ato B whose range
is a singleton then find (m + n).

 d2y
Q.200 For the curve sin x + sin y = 1 lying on the first quadrant, if Lim x · exists and has the non-zero
x 0 dx 2
2

value equal to L, find the value of   .
L

x
Q.201 Find the area enclosed by the curve | x – 60 | + | y | = .
4

tan 1 3
Q.202 If the value of  ln(1  3 tan x))dx = (arc tan a) · ln b where b  N, then find the value of (a + b).
0

Q.203 Let z and  be complex numbers such that z +  = i and z2 + 2 = 1. If area of triangle formed by
z,  and origin is equal toA, then find the value of 80A2.
[Note: i2 = – 1. ]

  
Q.204 If a , b and c are non - zero and non-coplanar vectors such that
              
[(a  b)  ( b  c ) ( b  c )  c  a  c  a   (a  b)]  k[ a b c ] 2 then find k.

e
t
Q.205 Let f (x) is a derivable function satisfying f (x) = sin( x  t ) dt and g (x) = f '' (x) – f (x).
0
If the range of g (x) is [a, b], then find the value of (a + b2).
2

Q.206 The roots of the equation z4 + az3 + (12 + 9i) z2 + bz = 0 are the vertices of a square then find
the value of (2b – 13a).

Q.207 Give that S = x 2  4 x  5  x 2  2 x  5 for all real values of x, find the maximum value of S4.

FINAL STEP Page # 30


MATHEMATICS
1
2  3x  4x 2
Q.208 Let  = 2 2
x  x 1
, then find the value of 2.
0

Q.209 Find the number of function f : {1, 2, 3, 4, 5}  {1, 2, 3, 4, 5} which assumes exactly 3 distinct values.

Q.210 If f(x) = x3 – 3x + 1, then find the number of distinct real roots of the equation f f ( x )   0 .

  f (t )  x 
2
Q.211 Let f(x) be a non-constant function f(x) such that f(x) = x2 – dt .
0
Compute the value of f (3).

Q.212 A region S in complex plane is S = {x + iy, – 1  x  1 and – 1  y  1}. A complex number


z = x + iy is chosen uniformly at random from S. If the probability P that the complex number
3 m
(1  i)z is also in S can be expressed in lowest form as , then find (m + n).
4 n

Q.213 The triangle ABC, right angled at C, has median AD, BE and CF. AD lies along the line y = x + 3,
BE lies along the line y = 2x + 4. If the length of the hypotenuse is 60, find the area of the triangleABC
(in sq. units).

Q.214 Let M be the set of all complex numbers z that satisfy | z – 4 – 3i | = r. If r is a positive constant such that
there is only one real number which belongs to M. For the value of r, find the maximum value of | z |.

 
sin x 2 2
Q.215 Let f be a continuous such that f(x) = sin3x +
  f ( x) sin x dx . If  =  f ( x ) dx ,
0 0

find the value of (12 ).

ln x
Q.216 If the curves C1 : y = and C2 : y = x2 (where  is constant) touches each other, then find the
x
reciprocal of the area bounded by curves C1, C2 and x-axis.

Q.217 Let z be a complex number satisfying | 2z + 3i| = |z2|. If M and m are the maximum and minimum
values of z, then find the value of (M + m).

Q.218 The parabola y = x2 is reflected in the line y = 3 producing a new parabola. This new parabola is
reflected in the line x = 2, producing y = x2 + bx + c. Find the value of (b – c).


 
 
 

   1     1   
Q.219 If a  b  b  c  b , a · b = = b · c , a · c = then find the value of 92 (where a , b, c are
2 3
unit vectors).

Q.220 The triangle formed by the normal to the curve f(x) = x2 – ax + 2a at P(2, 4) and the co-ordinate axes
lies in second quadrant. If its area is 2, then find possible values of a.

FINAL STEP Page # 31


MATHEMATICS
 
Q.221 Find the length of intercept on the normal at N 12, 8 3 on the parabola y2 = 16 x made by the circle
which is described on the focal distance of this point as diameter.

Q.222 Let z1, z2, z3 be complex numbers such that


z12 z2 z2
| z1 | = | z2 | = | z3 | = 1 and  2  3 1 = 0
z 2 z 3 z1z 3 z1z 2
Find the number of elements in the range of | z1 + z2 + z3 | .

Q.223 Let f = x4 + ax3 + bx2 + cx + d be a polynomial with real coefficients and real roots. If | f(i) | = 1,
then find the value of (a + b + c + d).


Q.224 A line L1 parallel to 3î  2ˆj  4k̂ passes through A a   7 î  6ˆj  2k̂ and another line L2 parallel to


2î  ˆj  3k̂ passes through B b  5î  3ˆj  4k̂ . If line L3 perpendicular to 2î  2 ĵ  k̂ intersects L1


and L2 at Aa  and C respectively, then find the value of | AC | .
[Note : [y] denotes greatest integer less than or equal to y.]

Q.225 Let f(x) = max.  5  x , 7 . If  is the number of points where f(x) is discontinuous
 x

and  be the number of points where f(x) is non-differentiable, then find ( + ).

FINAL STEP Page # 32


MATHEMATICS
ANSWER KEY
Q.1 B Q.2 C Q.3 C Q.4 A Q.5 B

Q.6 D Q.7 B Q.8 D Q.9 A Q.10 A

Q.11 C Q.12 B Q.13 C Q.14 C Q.15 D

Q.16 D Q.17 C Q.18 C Q.19 D Q.20 B

Q.21 C Q.22 C Q.23 A Q.24 D Q.25 C

Q.26 C Q.27 B Q.28 A Q.29 B Q.30 B

Q.31 D Q.32 A Q.33 A Q.34 A Q.35 C

Q.36 A Q.37 B Q.38 B Q.39 A Q.40 B

Q.41 C Q.42 D Q.43 D Q.44 C Q.45 A

Q.46 C Q.47 B Q.48 B Q.49 A Q.50 C

Q.51 C Q.52 C Q.53 B Q.54 C Q.55 B

Q.56 D Q.57 D Q.58 A Q.59 A Q.60 A

Q.61 D Q.62 C Q.63 D Q.64 A Q.65 A

Q.66 D Q.67 A Q.68 D Q.69 A Q.70 B

Q.71 B Q.72 D Q.73 C Q.74 A Q.75 C

Q.76 D Q.77 B Q.78 C Q.79 D Q.80 C

Q.81 B Q.82 C Q.83 B Q.84 D Q.85 B

Q.86 C Q.87 A Q.88 B Q.89 C Q.90 A

Q.91 C Q.92 A Q.93 D Q.94 D Q.95 C

Q.96 A Q.97 D Q.98 B Q.99 D Q.100 A

Q.101 D Q.102 A Q.103 B Q.104 C Q.105 D

Q.106 A Q.107 C Q.108 C Q.109 A Q.110 A

Q.111 AB Q.112 AD Q.113 ABD Q.114 BCD Q.115 BCD

Q.116 BCD Q.117 ABD Q.118 ACD Q.119 AD Q.120 AD

Q.121 ABCD Q.122 ABCD Q.123 BCD Q.124 AC Q.125 ACD


FINAL STEP Page # 33
MATHEMATICS
Q.126 ABC Q.127 ABC Q.128 BD Q.129 AD Q.130ABCD

Q.131 AC Q.132 CD Q.133 ABCD Q.134 AC Q.135 BD

Q.136 BC Q.137 ABCD Q.138 AC Q.139 AB Q.140 AD

Q.141 (A) S ; (B) Q ; (C) R Q.142 (A) Q ; (B) S ; (C) R

Q.143 (A) S ; (B) P, S ; (C) P, Q ; (D) P, Q, R, S, T Q.144 (A) Q, (B) R, (C) P

Q.145 (A) S, (B) P, Q, (C) R Q.146 (A) Q, (B) S, (C) T, (D) P

Q.147 (A) S ; (B) T ; (C) Q ; (D) P ; (E) R Q.148 (A) Q; (B) P; (C) Q; (D) P, Q, R, T

Q.149 (A) P ; (B) T ; (C) R ; (D) S Q.150 (A) R ; (B) P; (C) Q; (D) T

Q.151 0006 Q.152 0009 Q.153 0016 Q.154 0036 Q.155 0002

Q.156 0035 Q.157 0006 Q.158 0005 Q.159 0006 Q.160 0019

Q.161 0005 Q.162 0005 Q.163 0006 Q.164 0011 Q.165 0041

Q.166 0008 Q.167 0040 Q.168 0081 Q.169 0055 Q.170 0004

Q.171 0201 Q.172 0005 Q.173 0004 Q.174 0009 Q.175 0017

Q.176 0336 Q.177 0004 Q.178 0009 Q.179 1560 Q.180 0002

Q.181 0006 Q.182 0006 Q.183 0010 Q.184 0007 Q.185 0001

Q.186 0002 Q.187 0019 Q.188 0000 Q.189 0002 Q.190 0003

Q.191 0005 Q.192 0018 Q.193 0007 Q.194 0197 Q.195 0007

Q.196 0006 Q.197 0219 Q.198 0008 Q.199 0788 Q.200 0018

Q.201 0480 Q.202 0013 Q.203 0015 Q.204 0004 Q.205 0004

Q.206 0060 Q.207 0004 Q.208 0003 Q.209 1500 Q.210 0007

Q.211 0006 Q.212 0016 Q.213 0400 Q.214 0008 Q.215 0011

Q.216 0018 Q.217 0004 Q.218 0018 Q.219 0005 Q.220 0005

Q.221 0008 Q.222 0002 Q.223 0000 Q.224 0004 Q.225 0002

FINAL STEP Page # 34


MATHEMATICS
[HINTS & SOLUTIONS]

2 sin  cos(  ) sin( 2  )  sin  sin(2  )  sin   2 sin 


1. E= = =
2 cos  sin(  ) sin( 2  )  sin  sin(2  )  sin 
2 sin 
= 1
sin(2  )  sin 
  
For maximum E, 2 +  =  =   
2  4 2
2 sin  1  sin 
 Emax. = 1 – =  (B) Ans.
1  sin  1  sin 

2. P1, P2, ...... P7 = 7


P9, P10, ...... P16 = 8
—————————
8
C3 8·7·6 8
15 = = . Ans.
C3 15 · 14 · 13 65

3. Let 3  3  3  ....... = x

 3  x = x  3 + x = x2  x2 – x – 3 = 0
 P(x) = x2 – x – 3  P(5) = 25 – 5 – 3 = 17. Ans.

1 1 1 1 1 1 1
4.      
sin 2 sin 4 sin 8 sin 16 sin 32 sin 64 sin 128

sin 1 sin 2 sin 64


 ......................
sin 1sin 2 sin 2 sin 4 sin 64 sin 128

sin( 2  1) sin 2 cos 1  cos 2 sin 1


T1 = = =cot1 – cot2
sin 1sin 2 sin 1sin 2

sin(4  2) sin 4 cos 2  cos 4 sin 2


T2 = = =cot2 – cot4
sin 2 sin 4 sin 2 sin 4

sin(8  4) sin 8 cos 4  cos 8 sin 4


T2 = = =cot4 – cot8
sin 4 sin 8 sin 4 sin 8

sin(128  64) sin 128 cos 64  cos 128 sin 64


T7 = = =cot64 – cot128
sin 64 sin 128 sin 128 sin 64
sin = cot1 – cot128
cos1 cos 128 cos 1sin 128  sin 1 cos 128 sin 127
S=  = = = sin 127cosec1 cosec128
sin 1 sin 128 sin 1sin 128 sin 1sin 128

FINAL STEP Page # 35


MATHEMATICS
l1  l2  l3 4
5. If l1, l2 and l3 are the length of the medians and l = , then  = l (l  l1 ) (l  l2 ) (l  l3 )
2 3
Here, l1 = 3 ; l2 = 6 ; l3 = x
9 x
 l=
2
9 x 3 x
l – l1 = –3=
2 2
9 x x 3
l – l2 = –6=
2 2
9 x 9x
l – l3 = –x=
2 2

9  x  x  3  x  3   9  x 
4
 3 15 =      9 15 = (81  x 2 ) ( x 2  9)
3
2  2  2   2 
(81) (15) = (81 – t) (t – 9)  (81) (15) = 90t – t2 – 81 · 9  t2 – 54t – 36t + 54 · 36 = 0
 t = 54 or t = 36
 x2 = 54 or x2 = 36
 x = 3 6 or x = 6
but x = 6 is to be rejected as sides of the triangle are unequal  medians must be unequal.]

2
6. a1 = 0.6 = =a
3
a3 = 0.074
1000a3 = 74.074

74 37 · 2 2
999a3 = 74  a3 = =  a=
999 37 · 27 27

2 2 2 1
ar2 = ; r3 =  r2 =
27 3 27 27
1 1
 r2 =  r=
9 3
2 2 2
 G.P. is , , , ……
3 9 27

a
r 1
2
r = a12  a 22  a 32   

2 2 2
2 2  2 
=          
 3   9   27 
4/9 1
= = ]
1 1/ 9 2

FINAL STEP Page # 36


MATHEMATICS
BD AB B
7. =
DC AC
 AD is the angle bisector D
a=4
c=3
2bc A 2  5  3 1  cos A
 AD = cos =
bc 2 8 2
A b=5 C
3
1
15 5 15 2 3 5  5 3 5
= = × = = ]
4 2 4 5 2 5 2

 (n  1) (n  2) 2  2x 
8. As, Tn = cot–1  x    Tn = tan–1  
2 x  (n  2) (n  1) x 2  4 
  

 n  2   1   n  1  
 Tn = tan–1    x   tan    x 
 2    2  
 n  2   1
So, Sn = tan–1    x   tan x
 2  

 Lim Sn   tan 1 x  cot 1 x = 1 (Given)  x = cot 1. Ans.
n 2

9.    
P A B = P A c  Bc = P (A or B) = P (A  B)

 
P A c  Bc = P (A  B)

    1 2 7
 P A B  P A  B = P(A  B) + P(A  B) = P(A) + P(B) =  = .
2 3 6
A  B = {a1 + a2 + a3} A B
A  B = {a1 + a2 + a4} a1 a2 a3

A  B = {a2 + a3 + a4} a4

(A  B)  (A  B ) = {a1 + a2}
1 2 1
(a1 + a2)  (a2 + a3 + a4) = a2 = P (A  B) =   =  A. Ans.
2 3 3

10. Putting x = a and y = c


c = (m2 + 4) a2 + (m – 2)2 a – 4m + 2
c = (a2 + a) m2 – 4(a + 1)m + 4a2 + 4a + 2 [must be equal to c]
 (a + a)m – 4(a + 1)m + (4a + 4a + 2) – c = 0 must be an identity is m.
2 2 2

Hence a2 + a = 0 and a = –1
 a = –1 and c = 4a2 + 4a + 2 = 4 – 4 + 2 = 2
 a = –1; c = 2  (– 1, 2) Ans.

FINAL STEP Page # 37


MATHEMATICS
11. x = – 5 + 14 cos 
y = 12 + 14 sin 
———————
x2 + y2 = 365 – 140 cos  + 336 sin 
= 365 + 14 (24 sin  – 10 cos )
= 365 + 28 (12 sin  – 5 cos )
= 365 + 28 · (– 13) [Minimum value of (x2 + y2)]
x2 + y2 = 365 – 364 = 1. Ans.

1
12. Given, logxy =
4
y = x–1/4  yx1/4 = 1  xy4 = 1 .......(1)
 1  1  1 1 1 33
Now, E = logx(xy4y) – 2 log y x   = logxy –  8  = 8 = 8 + = . Ans.
 2  2  4 2 4 4

13. Given, a(5 sin2 – 11) = – sin2 + 8 sin  – 9


 sin 2   8 sin   9 sin 2   8 sin   9 (sin   4) 2  7
 a = = =
5 sin 2   11 11  5 sin 2  11  5 sin 2 
The above fraction will be maximum when numerator is maximum and denominator is minimum. It will

happen at  = 2n – , n  I.
2
(1  4)2  7 18
 Greatest value of a = = = 3 Ans.
11  5 6

6G 6 C
14. Lot A from A
4D New 1 item is taken
Lot
7G 4 of
Lot B from B 10
3D articles A E

3 4 2 B
A : item takes from A  P(A) = ; P(B) = =
5 10 5
E : item drawn from mixed lot is defective
P(E) = P(E  A) + P(E  B)
= P(A) + P(B) · P(E/B)
3 2 2 3 6 3 9
= ·  · =  = . Ans.
5 5 5 10 25 25 25

FINAL STEP Page # 38


MATHEMATICS
15. Using cosin rule in ABC
a 2  b2  c2
we get, cos C =
2ab

2 81  64  x 2
 =
3 2(9)(8)
 96 =145 – x2
 x2 = 49  x = 7 Ans.

y
16. Let f(x) = ( x  3) ( x  1)
for 3 solution
3
y = mx – 2m must be tangent to
y = – (x2 + 4 + 3); x  (– 3, – 1) 2
 2
x + x (4 + m) + 3 – 2m = 0 1
D=0
(4 + m)2 – 4 (3 – 2m) = 0 x
16 + m2 + 8m – 12 + 8m = 0 –3 –2 –1
m2 + 16m + 4 = 0
 16  2 60
m= = – 8  60 (+) reject
2
 m = – 8  60 ]

17.  (1  x )  (1  x ) 2  .......  (1  x )16


Let E = 1
Geometric progression
1st term 1, common ratio (1 x )
total number of terms 17

 (1  x )17  1   (1  x )17  1 
= 1·   =
 


 (1  x )  1   x 
 Coefficient of x from E = Ck + 1 and coefficient of x2k from E = 17C2k+1
k 17

As, 17C 17
k + 1 = C2k + 1 (Given)
 Either (k + 1) = (2k + 1)  k = 0 (rejected as k  N)
or (k + 1) + (2k + 1) = 17  3k = 15  k = 5 Ans.

18. Note that 5  2 is a positive proper fraction.


  5 2  p
is also a positive proper fraction say f '.

Now consider  52  –


p
5 2  p
= 2  C1 5

p
  p 1
· 2  p C3 5   p 3
· 23  .......  p C p · 2 p 

p  1 
is even 

I + f – f ' = 2  p C1 5

  p 1
· 2  p C3 5   p 3
· 23  .......  p C p · 2 p 


{I is the integral part and f is the fractional part of  52  p


}
Note that – 1 < f – f ' < 1  f–f'=0



I – 2p + 1 = 2p  5   p1
·2 
(p  1)(p  2)
3!
 5 3
· 23  .......  p C p 2  5 ·22 p 2 

 
FINAL STEP Page # 39
MATHEMATICS



p
but I =  2  5 


p


p 1

Hence  2  5   (2) = 2p (an integer) must be divisible by 2p. Ans.

19. In ABC, we have, r : b + c : a = 2 : 17 : 13


Let r = 2, b + c = 17 , a = 13(where  > 0)
a  b  c 17  13
 s= = = 15 and (s – a) = (15 – 13) = 2
2 2
A A
Now, r = (s – a) tan  2 = 2 tan  A = 90°. Ans.
2 2


20. We have 2 
3
2

 2 tan 1 x 2  x  k   1 1 
 As, tan   cot      R 
 2 

 tan–1 (x2 + x + k) =  x2 + x + k = 1  x2 + x + (k – 1) = 0
4
 For required condition, put D > 0
5
 1 – 4(k – 1) > 0  5 – 4k > 0  k < . Ans.
4

r · 100 C r r ·100 ! (r  1) !(101  r ) !


21. = · = 101 – r
100
C r 1 r !(100  r )! 100 !
100
 S=  (101  r) = 100 + 99 + 98 + ....... + 1 = 5050. Ans.
r 1

22. Let the circle be x2 + y2 + 2gx + 2fy + c = 0 as center lies on 2x – 2y + 9 = 0


 – 2g + 2f + g = 0
As it cuts x2 + y2 = 4 orthoganally
 2g × 0 + 2f × 0 = c – 4
 c=4
equation of circle x2 + y2 + (2f + 9)x + 2fy + 4 = 0
(x2 + y2 + 9x + 4) + 2f (x + y) = 0
it passes through point of intersection of x2 + y2 + 9x + 4 = 0 and x + y = 0
 1 1 
 points  ,  and (– 4, 4)
 2 2
1
a= c=–4
2
1
b= d=4
2
4a2 + 4b2 + c2 + d2
1 1
4 × + 4 × + 16 + 16
4 4
1 + 1 + 32 = 34

FINAL STEP Page # 40


MATHEMATICS

23.

 k  5  k  1 
We have    =–1
 h  1  h  2 
 Locus of (h, k) is x2 + y2 – 3x + 4y – 3 = 0  (C)

24. We have f ''(x) = 6(x – 1)


Integrating, we get f '(x) = 3(x – 1)2 + c ... (1)
At (2, 1), y = 3x – 5 is tangent to y = f '(x)
 f '(2) = 3
From (1), 3 = 3(2 – 1)2 + c  3 = 3 + c  c = 0
 f ' (x) = 3(x – 1)2
Since the curve passes through (2, 1)
 1 = (2 – 1)3 + c'  c' = 0
 f(x) = (x – 1)3

 ln (1  tan a · tan x) dx 0


25. Lim 0   form
a 0 a3 0
Note that Lopital rule with Lebnitz cannot be applied. (think!)
a
Let I =  ln (1  tan a · tan x ) dx ......(1)
0

a a
 tan a  tan x 
I =  ln (1  tan x · tan (a  x ) dx =  ln 1  tan a ·  dx
0 0  1  tan x · tan a 
a
 1  tan a · tan x  tan 2 a  tan a· tan x 
I =  ln   dx

0  1  tan x · tan a 

 
a
I =  ln ln (1  tan 2 a )  ln (1  tan a · tan x ) dx ......(2)
0
Adding (1) + (2)
a
2I =  ln (1  tan 2 a ) dx = a ln (1 + tan2a)
0

a ln (1  tan 2 a )
I=
2
Now, we have
a
ln (1  tan 2 a ) · tan 2 a
tan 2 a 1
Lim 2 3 2 = Lim = . Ans.
a 0 a · tan a a 0 2a 2 2

FINAL STEP Page # 41


MATHEMATICS
a

 ln (1  tan a · tan x) dx
Aliter: Lim 0
a 0 a3
a
Nr : I =  ln (1  tan a · tan x ) dx
0
 King and add
a
2I = ln(1 + tan2 a)  dx  a ln (1  tan 2 a )
0

a
 I= ln(1 + tan2 a)
2
a
ln (1  tan 2 a ) 1
Hence, Lim 2 = . Ans.
a 0 a3 2

26. O, A, B and D are concyclic Im

BD 3
cos 60° = 
AD AD B(z2)
D 60°
(5i)
AD = 6 and OD = 5 (z1)
C
60°
z 3  OA  36  25  11 . Ans. O A(z3)
Re

27. P(x) = k(x – 2)2 (x – 3)2 + 3


P(1) = 4k + 3 = 7
 k = 1  P(x) = (x – 2)2 (x – 3)2 + 3
 P(5) = 9 × 4 + 3 = 39. Ans.

28. Differentiate w.r.t. x


y ' (x) + y(x) = 2x
dy
+ y = 2x
dx
I.F. = ex
yex = 2  xe x dx


yex = 2 xex  e x  C 
y = 2(x – 1) + ce–x
at x = 1 ; y (1) = 1  c = e
y = 2(x – 1) + e1 – x. Ans.

FINAL STEP Page # 42


MATHEMATICS
293
29. d= ]
7

1 i 3 1 i 3
30. z2 – z + 1 = 0  z = or
2 2
Hence, z = – w or z = – w 2

z 2  z  1) (z 2  1)  z
Now, z4 – z3 + 2z2 – 2z + 1 = z2 (z2 – z + 1) + 1 (z2 – z + 1) – z = (   
vanishes
Hence, E = ( – z1 )2013 + (– z2 )2013
2013 2013
     
 E =  cos  i sin    cos  i sin 
 3 3 3 3
 
z z

2013

– E = 2Re z = 2 cos    – E = – 2  E = 2. Ans.
3

31. The line is (x – 2y –1) +  (x + y –2) = 0 ....(1)


 x (1 + ) + y (– 2) – (1 + 2) = 0 L1 L2
For the circle Centre  (3, –1)
L
Radius = 9  1  6 = 2 2
(3,–1)
 , distance of line from the centre of circle is equal to radius.
3(1   )  (  2)  (1  2 )
 =2
(1   ) 2  (  2) 2
 (3 + 3 –  + 2 –1 – 2)2 = 4[1 +2 + 2 + 2 – 4 + 4]
 42 = 4 (22 – 2 + 5)  22 – 2 + 1 = 0
 D < 0  No real 
 No such line exists
Alternate : - L1 : x – 2y = 1 .....(1)
L2 : x + y = 2 .....(2)
5 1
Point of intersection :  ,  which lies inside the circle
 3 3
2 2
5 1 5 1 4
 S1 =      – 6     2     6 = <0
 3 3 3  3 9
There can’t be a tangent through an interior point of circle.

FINAL STEP Page # 43


MATHEMATICS
cos x x 1
32. Given, g(x) = 2 sin x x2 2x
tan x x 1
= cos x (x2 – 2x2) – x (2 sin x – 2 x tan x) + 1 (2 x sin x – x2 · tan x)
= – x2 · cos x + 2 x2 · tan x – x2 · tan x
= – x2 · cos x + x2 tan x
 g(x) = x2 (tan x – cos x)
Also, g(0) = 0 and g '(x) = x2 (sec2x + sin x) + 2x · (tan x – cos x)
 g '(0) = 0
So, equation of normal drawn to g(x) at x = 0, is x = 0. Ans.

Q.33 (1  i) 
2 50 = (2i)50 = (2)50 (i2) = – 250 Ans.

Q.34 SS' = 2ae, where a and e are length of semi-major axis and eccentricity respectively.
 (9  3) 2  (12  4) 2  2ae
 ae = 5
 centre is mid-point of SS'
 centre  (6, 8)
Let the equation of auxiliary circle be (x – 6)2 + (y – 8)2 = a2
We know that the foot of the perpendicular from the focus on any tangent lies on the auxiliary circle
 (1, – 4) lies on auxiliary circle
i.e. (1 – 6)2 + (–4 – 8)2 = a2  a = 13
 ae = 5  e = 5/13 Ans.

 
1
1  tan 2 ln (1  x ) ln (1 x )
Q.35 Using L' Hospital's rule Lim
x 0 (1  x ) · cos x

 
1
L = Lim 1  tan 2  ln (1  x )  ln (1 x ) = el
x 0

1  2 tan  2  ln (1  x )  
where l = Lim  tan  2 ln (1  0)   = =–2
x 0 ln (1  x ) 2ln (1  x)
1
 L = e–2 =  a = 2. Ans.
e2

y1
Q.36 mOP =
x1

y1 = 2x1  3

dy 1
=
dx 2x  3

FINAL STEP Page # 44


MATHEMATICS
dy 1
 =
dx x1 , y1 2x1  3

1 y 2 x1  3
 = 1 =
2x1  3 x1 x1
y
2x1 – 3 = x1  x1 = 3
(3, 3 )
3 P(x1,y1)
3· 3
Required area =
2
 3 2 x  3 dx
3
2

3
3 3 2 (2 x1  3)3 2  O0 1 3 2 N (3,0)
=   2
2 3 2 3 / 2

3 3
2

1
3
3 30  
3 3 3
 3 =  (A). Ans.
2 2

a+1
37. f '(x) = 3x2 – 6ax + 3(a2 + 1) = 3 (x2 – 2ax + a2 – 1)
a–1
 m = x1 = a + 1 (point of maxima)
M = x2 = a – 1 (point of minima)
M m
 a + 1 < 4 and a – 1 > – 2 graph of x v/s f '(x)
a < 3 and a > – 1 –2 4
a  (– 1, 3)  a2 – a1 = 4. Ans.  
(a – 1) (a + 1)

| z2 |
38. cos  =
| z1 |
y
Q(z2)
|z | P(z1)
Also, cos 2 = 3 R(z3)
| z2 |

cos  | z2 | 2
 = .......(A) 
cos 2 | z1 | | z 3 |
x
O
z2 z
= 1 ei .......(1)
| z 2 | | z1 |

z3 z
Also, = 2 ei .......(2)
| z3 | | z 2 |
 (1) × (2)
z 22 i z1z3
2 e = ei
| z2 | | z1 | | z 3 |

z 22 | z2 | 2
 = .......(B)
z1z3 | z1 | | z 3 |
FINAL STEP Page # 45
MATHEMATICS
From (A) and (B)
z 22 cos 
=
z1z3 cos 2

z 22 2 3 2 3 cos  2  3 1  cos 2
Given =  =  =
z1z3 2 3 2 3 cos 2 2 3 cos 2

2
 = sec 2 2 = 30°  = 15°. Ans.
3

39.  
Add the two equations to get 8 x12  y12 = 40  x12  y12 = 5  r = 5  A ]

40. Let f(x) = ex + x – 2


f (0) = – 1 < 0 and f(1) = e – 1 > 0
Also f '(x) > 0
Hence,  exactly one root of f(x) in [0, 1] and no other root. Ans.

cot 1 
 tan x 
41. We have, f() =    dx
1  tan x  cot x 
... (1)
tan 

cot 1 
cot x
 f() =  cot x  tan x
dx ... (2)
tan 1 

(Using king property)


 Adding (1) and (2), we get

1 1  1  
f() = (cot–1  – tan–1 ) =   2 tan   = – tan–1
2 2 2  4

  3 
Clearly, range of f() is   ,  . Ans.
 4 4 

42. By using condition of tangency, we get 4h2 = 3k2 + 2


 Locus of P(h, k) is 4x2 – 3y2 = 2 (which is hyperbola.)
4 7
Hence e2 = 1 +  e= Ans.
3 3

1
5
 5 x 4 10 x 3 10 x 2  5 x 10
e
x
43. ( x  1)4 dx
1
Putting x5 – 5x4 + 10x3 – 10x2 + 5x + 10 = t
(5x4 – 20x3 + 30x2 – 20x + 5) dx = dt
1
5(x – 1)4 dx = dt  (x – 1)4 dx = dt
5

FINAL STEP Page # 46


MATHEMATICS

 
11
1 1 11  21 e32  1
I =  e dt = e  e
t
 = .
5 21 5 5e 21
 p + q = 32 + 2 = 53. Ans.

44. Put tan x = t


( t  4) dt dt ( t 2  4)  t 2
 I=  2
t ( t  4)
=  t2  4 +  t ( t 2  4)
dt

dt dt t dt
=  t2  4 +  t
–  t2  4
1 t 1
= tan 1  ln t  ln ( t 2  4)
2 2 2
1  tan x  1
= tan 1 2
  ln (tan x )  ln (tan x  4) . Ans.
2  2  2

n
1  k 2 2 k 
1
45. Lim     ln 1   =  [ x 2  ln 2 (1  x )] dx
k 1 n  n   n  0
n 

1
 + 2 (ln 2 – 1)2. Ans.
3

46. First equation is


(x – 6)2 + (y – 3)2 = 49 .......(1)
(x – 2)2 + (y – 6)2 = 40 + k .......(2)
C1 : Centre (6, 3) ; r1 = 7 P
(2,6)
C2 : Centre (2, 6) ; r2 = 40  k C2
The two circles will intersect is (6, 3)
C 2P  7 – 5 C1
40  k  2
k  – 36 Q
and C 2Q  7 + 5
40 + k  2
k  104
 – 36  k  104
 (b – a) = 140. Ans.

2 2
1  cos x
 sec x  tan x dx =  ln (tan x  sec x)  ln (sec x) 0
2
47.  cos x
dx =
0 0

=  ln (sin x  1)  0
2
= ln 2. Ans.

FINAL STEP Page # 47


MATHEMATICS
48. Let n̂ = unit vector perpendicular to OA and BC O( 0 )
C( c )
  
a  (b  c)
=   
a  (b  c)
Now, shortest distance between OA and BC A( a )
B( b )

 
    

(b  a ) · a  (b  c)
  
 
  
a b c 
= ( b  a ) · n̂ = a  (b  c ) =   
a a  c sin 

where  = angle between O A and B C

6V
= = V = 12 Ans. (As volume of tetrahedron = 12 (Given) ]

3(4) sin
6

2
( x  1) e 2 x 3
49. Let I =  1  (x  1) (e2 x 3  1) dx
1

1
Put 2x – 3 = t  2 dx = dt  dx = dt
2
1 1
1 ( t  1) e t 1  ( t  1) e t ( t  1) e  t 
So, I=  t
dt =    
 2  ( t  1) (e t  1) 2  ( t  1) (e  t  1)  dt
2 1 2  ( t  1) (e  1) 2 0  
1 1
1  ( t  1) e t (1  t )  1  ( t  1) e t  (1  t ) 
=
2   ( t  1)et  (1  t )  ( t  1) e t  (1  t )  dt = 2
   (t  1)e t  (1  t)  dt
0   0  
1
1 1
= 
2 0
dt = . Ans.
2

50. Centres are (10, 0) and (– 15, 0)


r1 = 6; r2 = 9
d = 25
r1 + r2 < d
 circles are separated
PQ = l = d 2  (r1  r2 ) 2

= 625  225 = 20 Ans.

t
2
51. sin t dt = f(t) + C
 f '(t) = t2 sin t
[f (3x  3h )  f ( x  h )]  [f (3x )  f ( x )]
 Given limit is Lim
h 0 h
[f (3x  3h )  f ( x  h )]  [f (3x )  f ( x )]
Lim
h 0 h
3 f '(x) – f '(x) = 3 (9x2 sin 3x – x2 sin x) = x2 (27 sin 3x – x2 sin x). Ans.
FINAL STEP Page # 48
MATHEMATICS

î ĵ k̂
52. Normal vector of required plane is parallel to vector = 2 1 1
1 2 1

= î (1  2)  ˆj( 2  1)  k̂ ( 4  1) =  î  3ˆj  5k̂


 Equation of required plane is
– 1(x + 1) + 3(y – 3) + 5(z – 2) = 0 or x – 3y – 5z + 20 = 0. Ans.

6 6 6 6
1  sin x 1  sin x 1  sin x dx dx
53. I=  cos x
dx =  cos x 1  sin x
dx =  1  sin x
=   x x
0 0 0 0  cos  sin 
 2 2

x
Put =t
2
 12  12  12
dt dt  
=2  (cos t  sin t )
= 2   
= 2  sec   t  dt
4 
0 0 cos   t  0
4 
 12
             
= 2 ln  sec   t   tan   t   = 2 ln  sec  tan   ln  sec  tan 
 4   4  0   3 3  4 4 

=  
2 ln 2  3  ln   2 1  =  2 3 
2 ln 
2  1
 . Ans.

 

(a  1)3 (a  2)3 (a  3)3


54. We must have  = 0 i.e. (a  1) (a  2) (a  3) = 0
1 1 1

Put u = a + 1, v = a + 2, w = a + 3
then u – v = – 1, v – w = – 1, w – u = 2
Also, u + v + w = 3a + 6
Now,  = 0

u3 v3 w3
 u v w =0
1 1 1

i.e., (u – v) (v – w)(w – u)(u + v + w) = 0  (–1) (–1) (2) (3a + 6) = 0


or a = – 2 Ans.

FINAL STEP Page # 49


MATHEMATICS
 z  z1 
55. Given, arg  = tan–13 = 1 P(z)

 z  z 2 

5
 From figure sin  =
2r C
r 
3 5 5
 = 5
10 2r z1
2
2
(–3, 2) z2
5 5 (2, –3)
 r= . Ans.
3

FINAL STEP Page # 50


MATHEMATICS
Paragraph for question nos. 56 & 57
A: 2 items (out of 4 chosen for inspection) found to be both good.
3
C3 · 6C1 6
B1 : 3D and 1G drawn ; P(B1) = 9 =
C4 126
3
C 2 · 6C 2 45
B2 : 2D and 2G drawn ; P(B2) = 9 =
C4 126
3
C1 · 6C3 60
B3 : 1D and 3G drawn ; P(B3) = 9 =
C4 126
6
C4 15
B4 : All 4G drawn ; P (B4) = 9 =
C4 126
2 3 4
1 C2 C23 C2 6
P(A/B1) = 0; P(A/B2) = 4 = ; P(A/B3) = 4 = ; P(A/B4) = 4 =
C2 6 C2 6 C2 6

P( B1 / A) = 0;
 
E1
45 1
P(B 2 ) · P(A / B 2 )  45 45
P (B2 / A ) = 4 = 126 6 = =
  45 1 60 3 15 6 45  180  90 315
E2  P(Bi ) P(A / Bi ) 0  126  6  126  6  126  6
i 1
180 90 2
Similarly, P(
B3 / A) =
  315 ;
P (B4 / A ) =
 =
 315 7 Ans.(i)
E3 E4

E: Next drawn item is good


P(E) = P(E  E2) + P(E  E3) + P(E  E4)
= P(E2) · P(E/E2) + P(E3) · P(E/E3) + P(E4) · P(E/E4)
180 1 90 2
= 0 + · + ·
315 2 315 2
90 90 180 36 4
=  = = = Ans.(ii)
315 315 315 63 7
Aliter: It is given that first two draws have resulted into good items. Now remaining items are 4G and 3D.
4
Next item drawn to be good, then probability = Ans.
7

Paragraph for Question Nos. 58 to 60


(–8, –4, 3)
L2 Z (Normal plane)
S
R P(1, 2, 3)

T2
Q T1

Plane P1

FINAL STEP Page # 51


MATHEMATICS
x 1 y  2 z  3
  =  so Q(x, y, z)  (3 + 1, 2 + 2, + 3)
3 2 1
z = 0, so = –3 thus Q(x, y, z)  (–8, –4, 0)
Since, L2 is reflected light so we can find a point ‘R’.
so that RQ and PQ are equal
here S = (–8, –4, 3)
so point R(a, b, c) (let)
a 1 b2
= –8 so a = – 17, = – 4 so b = – 10
2 2
R(a, b, c)  (–17, –10, 3).
  
RQ  OQ  OR = (8  17)iˆ  ( 4  10)ˆj  (0  3)kˆ
= 9iˆ  6ˆj  3kˆ  3(3iˆ  2ˆj  k)
ˆ

Thus line L2; r  ( 8iˆ  4ˆj)   (3iˆ  2ˆj  k)
ˆ

ˆi ˆj kˆ
  ˆi(2  2)  ˆj(3  3)  k(6
ˆ  6)
r3 2 1
3 2 1  4iˆ  6ˆj  2(2iˆ  3j)
ˆ

   
(r  a).n  0 so (r  ( 8iˆ  4ˆj)).( 2iˆ  3j)
ˆ 0
–2x + 3y – (16 – 12) = 0
so –2x + 3y – 4 = 0 or 2x – 3y + 4 = 0 & z = 0
Equation of plane bisecting 2x – 3y + 4 = 0 & z = 0
(2x  3y  4) z

13 1
2x – 3y  13 z + 4 = 0

Paragraph for Question Nos. 61 to 63

Since f(x) is symmetric about x = 1 and it is twice differntiable. y


so f '(x) must have one root at x = 1.
f '(x) = a(x – 1) (x – ) (x – ) x
O x=
= a(x – 1) (x2 – ( + ) x + ) x=
x=1
 
Here 1 so +=2
2
f ' (x) = a(x – 1) (x2 – 2x + )
= a(x3 – 2x2 +  x – x2 + 2x – 
f " (x) = a(3x 2 – 6x +  + 2)
f "(2) = a(12 – 12 +  + 2) = 0
so  = – 2
2 2
– 2 so   2  2  0

  1 3 ,   1 3
so sum = 1 +  +  = 3
f ' (x) = a (x – 1) (x2 – 2x – 2) = a(x –1) ((x– 1)2 – 3)

FINAL STEP Page # 52


MATHEMATICS
  x  14 3 

f(x) = a  4   x  1 2
C

2
 

 f (1)  0 so c  0 
 4
 f (2)  1 so a   5 

1 3 4
f(2) = a     1 so a = –
4 2 5

4   x  1 3 
4
  x  1 
2
f(x) =  
5 4 2 

4 16 3 2 4 8
f (3) =     2      4  6  
5 4 2  5 5
4 9 3  4  9 9
f(1+ 3 ) =     3       
5 4 2  5  4 5

Paragraph for Question Nos. 64 & 65

1/ 2 1 3 1/ 2 1 y
y=x2
A= 0 x 2 dx =
3
[x ]0 
24 1 1
y=(x–1)2
 2, 2
 
1 1  3 1/ 2
 [x ]  [(x  1)3 ]1/ 2f ()  x
24 3 O (1, 0)
x=
3 x =+ f()
1 1 3 3  1
       f     1    
8 8  2
1
3     f     1 
3

8
f() +  – 1 = ( – 1/8)1/3
3

f() = (1 – ) + (3 – 1/8)1/3


f(x) = (1 – x) + (x3 – 1/8)1/3

Lt
 f  x   x  1
3

 Lt
x 3
 1/ 8   1 1
Lt  x 2  x  
x 1/ 2  1 x 1/ 2 (x  1/ 2) x 1/ 2  2 4
x 
 2

1 1 1 3
   
4 4 4 4
2 / 3
1 3 1
f ' (x) = 0 – 1 +  x  
3 8
 3x  2

2/3 2
 3 1  1
x    x2   x3    x6
 8  8

FINAL STEP Page # 53


MATHEMATICS
3 1 3 1
so x    x  2x3 =
8 8
1
so x3 = .
16

Paragraph for question nos. 66 to 68

x 2  ( y  2) 2 = y
Given, | z – 2i | = Im(z) or | x + i (y – 2) | = y or
 On squaring both sides, we get x2 + (y – 2)2 = y2  x2 – 4y + 4 = 0 or x 2 = 4 (y – 1) ....(1)
y

(0, 2)
S A (z0)
90°
x2 = 4(y – 1)

V (0, 1)

y=0
x
M O (Directrix)
(–1, 0)

(i) From equation (1), we conclude that locus of P(z) represents a parabola whose focus is (0, 2)
 (D) is correct.
(ii) From above figure, using property of parabola, ASM = 90°
 2i  z 0   We know that portion of tangent
So, using rotation, we get arg  =
 2i  1  2 between point of contact and
directrix subtends right angle at
2i  z0 focus.
 is purely imaginary..
1  2i
2i  z 0 z0  2i
So,  = 0.  (A) is correct.
1  2i 2i  1
(iii) Using property of parabola, the equation of circle whose ends of diameter are z0 and 2i is
 z  2i  | z  2i |
z   0  = 0 , which always touches the tangent at vertex, which is y = 1, So Im(z)  1.
 2  2
 (D) is correct]

Paragraph for question nos. 69 & 70

(i) i = {1, 2, 3, 4} gives f (1) < 9 f (2) < 8 f (3) < 7 f (4) < 6
Thus, f (3) = 2 or 4 or 6  total number of possible functions for f : A B is
4 · 3 · 4 · 4 = 192 Ans.
(ii) i = {1, 2, 3, 4}
f (1)  2 f (2)  3 f (3)  4 f (4)  5
So the total number of possible functions is 3 · 3 · 4 · 4 = 144 Ans.

FINAL STEP Page # 54


MATHEMATICS
Paragraph for question nos. 71 & 72
C1 : (x – 1)2 + (y + 1)2 = 1 and C2 : (x – 1)2 + (y + 1)2 = 2 .
C2 is director circle of C1.
Since PAB is right angled triangle and APB = 90°. So, orthocentre of PAB is P and circumcentre
is midpoint of chordAB.

A
1
1 M P
C
(1,–1) 1
1
C1 B

2
 1 
Hence locus of M is (x – 1)2 + (y + 1)2 =   . Ans.
 2

Paragraph for Question no. 73 & 74


 z1  2 
Since Re z  2   0 z1
 1 
z1  2
 z1  2 is P.I. (–2, 0) 0 (2, 0)

 line joining (2, 0), z1 and (– 2, 0), z1 are perpendicular


 z1  2
Also, 2z22 – 2z1z2 + 3z12 = 0
z2
2t2 – 2t + 3 = 0 t= z
1

22 5i
t=
4

1 5 i
t=
2

1  5 i 
z2 =  2  z1
 

6
 z2  z1 .
2
z2 z
Also,  1 ei 
z2 z1 Q(z2) P(z1)

z2 z 
  2 ei 
z1 z1 O

FINAL STEP Page # 55


MATHEMATICS
1 5 i 6 i
= e
2 2

1 5
  i = cos  + i sin 
6 6
1
cos  =
6

5
sin  = ±
6

1 1 6 5
 OPQ = z1 z 2 sin  = z1 z1
2 2 2 6
Now, z1  2

1    
 D= 2   6  2   5   5 . Ans.
2  2   6

Paragraph for Question no. 75 to 77

(i) If y = mx + c is a common tangent, so


5
c= = ± 16m 2  9 = ± 29m 2  4  m2 = 1  m = ± 1
m
So, y = ± (x + 5)
(ii) Point of intersection of common tangents is P (– 5, 0).
Chord of contact from P to the parabola is
0 · y = 10(x – 5) x = 5
x (5)  16
to the ellipse is  0 = 1 x =
16 5
  16 9  y
So, the quadrilateral formed is trapezium.  ,  (5, 10)
1  41   18   5 5
Required area =    20  
2  5  5 x
O
P(–5, 0)
1  41   118  (41) (59)   16  9 
=    =  , 
2  5  5  25 (5, –10)
 5 5 
x2 y2
(iii) Ellipse touching x2 + y2 = 16 and x2 + y2 = 29 is  = 1 (As x-axis is major axis) ]
29 16

Paragraph for question nos. 78 to 81


f(x) = (x – 1) (x + 2) (x – 3) (x – 6) – 100
2 2
= ( x  4 x  3) ( x  4 x  12)  100
= (x2 – 4x)2 – 9(x2 – 4x) – 136
= (x2 – 4x – 17) (x2 – 4x + 8)

FINAL STEP Page # 56


MATHEMATICS
(i) f(x) = 0

( x 2  4x  17) ( x 2  4x  8)  0
 
D0 D0
 equation has two distinct real and two imaginary roots.
(ii) f(x) = (x2 – 4x – 17) (x2 – 4x + 8)
Since, f(x) is a polynomial function of even degree and defined from R to R.
 It is manyone into function.
(iii) f(x) = (x2 – 4x – 17) (x2 – 4x + 8)
= [(x – 2)2 – 21] [(x – 2)2 + 4]
f (x) min = (– 21) (4) = – 84 which occurs at x = 2.
g( x )
(iv)   f (x ) dx does not contain any logarithmic function.
g( x ) g( x ) Ax  B Cx  D
= 2 2 = 2  2
f (x ) ( x  4 x  17) ( x  4 x  8) x  4x  17 x  4x  8
Clearly, A, B and C must be zero.
g( x ) D
 2 2 = 2
( x  4 x  17) ( x  4 x  8) ( x  4 x  8)
2
g(x) = D (x – 4x – 17)
g(– 2) = D(4 + 8 – 17) = – 10
D · (– 5) = – 10
D = 2.
g( x ) 2( x 2  4 x  17) 2
 = 2 = 2
f (x ) 2
( x  4 x  17)( x  4 x  8) x  4x  8
4 4 4
g( x ) 2 dx
 f (x )
dx =  2
x  4x  8
dx = 2 
( x  2) 2  4
0 0 0

4
1  x  2 
= 2 ·  tan 1   = tan–1(1) – tan–1 (– 1)
2  2 

  
=   = . Ans.
4  4  2

FINAL STEP Page # 57


MATHEMATICS
Paragraph for Question no. 82 to 84

  2 
5x
 2 0 0
x  1
 0 0   x 1  5 0 0 
 x  3
AB =  0 x 0   0 0  = 0 3 0 
 0
x  0 0 4 
0 16   1  
   0 0
   4 

 1   1 
0 0  0 0 
 5 52
 1   1 
(AB)–1 =  0 0  , (AB)–2 =  0 0  and so on
 3   32 
 1  1 
 0 0  0 0 
4   42 

1 1 1 

 5 52  ...... 0 0 
 5n 
1 1 1
 X=  0  2  ...... n 0 
 3 3 3 
 1 1 1 1
 0 0    ........  n 
 4 4 2 43 4 
 1 
 5 
 1 0 0 
1   1 
 5  4 0 0
 1 
   1 
Y = Lim X   0 3 0   Y = 0 0
n 1  2 
 1  0 1
 3  0
 1   3 
 4 
 0 0
1
 1 
 4

4 0 0
 Y–1 = 0 2 0
 0 0 3
 
(i) adj  5 Y 1  =  5  adj Y 
2 1
= 53 adj Y
1
  = 53 · Y 1
2
= 53 · (4 · 2 · 3)2 = 5 · (5 !)2

 1  1  1  1 
x  x 0 0 4 0 0   x   0 0
4 x
 1   1   1

 0 0 0 0  0 0
(ii) AY =  x   2  =  2x 
 0 0 16 0 1  16 
0  0 0 
   3   3

FINAL STEP Page # 58


MATHEMATICS
1 1 1 16
 tr(AY) = x  
4 4x 2x 3
1 3 16 1 3 16 3 16
= x   2· ·x·     0.866 + 5.333  6.199
4 4x 3 4 4x 3 2 3
 Least integral value of tr (AY) = 7.
4 0 0 1 0 0  2 0 0
(iii) Z = Y–1– 2I = 0 2 0 – 2 0 1 0 = 0 0 0
   
 0 0 3 0 0 1  0 0 1
     
 tr (Z + Z + Z + ............ Z ) = 2 + 2 + 2 + ........... + 210 + 10
2 3 10 2 3

 210  1 
= 2 ·    10 = 211 + 8 = 2a + b

 2 1 
 a + b = 11 + 8 = 19. Ans.

Paragraph for question No. 85 to 87


(i) The system will have a unique solution if
1 1 1 1 1 1 1 1 1
1 2 3 0 or 0 1 2 0 or 0 1 2 = – 5 5]
1 3  0 1 3 0 0  5

LM 1 2 1 OP
(ii)
MM 1
For  = 5, adjoint of the coefficient matrix is given by 2 4 2
PP
N 2 1 Q
LM 1 2 1 OP LM5 OP LM  13 OP
we have (Adj. A) B = M2 4 2 P M9 P = M2  26P
MN 1 2 1 PQ MNPQ MN  13 PQ
The system will have no solution if  – 13  0 or  13 ]
(iii) The system has a infinite number of solutions if
= 5 and  = 13]

Paragraph for Question no. 88 to 90


P(x) = –x5 9x4
+ px3 – 27x2 + qx + r
 P(x) is divisible by x2
 q=r=0
P(x) = x5 – 9x4 + px 3 – 27x 2

P( x )
= x3 – 9x2 + px – 27 = 0 
x2

, ,  R+
 +  +  = 9,  = 27
  
A.M. of , ,  = =3
3

FINAL STEP Page # 59


MATHEMATICS
G.M. of , ,  = ()1/3 = (27)1/3 = 3
 A.M. = G.M. =  = 
 3 = 9  = 3 =  = 
p =  +  +  = 27

(i) p + q + r = 27 + 0 + 0 = 27

(ii)  – 1 = 2,  + 3 = 6,  + 7 = 10
 Sn = 2 + 6 + 10 + ....... to n terms = 2 (1 + 3 + 5 + ...... to n terms) = 2n2
   
1 1 1 1 1  1 1
Now,  =  2 2
=  n (n  1)
=
2
  n  1  n 
n 2 Sn · Sn 1 n 2 2n · 2 (n  1) 2 n 2 n 2

1 1 1 1 1  
=      .......  = . Ans.
2 1 2 2 3  2

 1 1 1 1 
(iii) Lim     ........  
n { q = r = 0}
n  27 ( 27) 2 ( 27 ) 3
( 27 )
 

1
1

1

1 27 = 1 . Ans.
= 2 3 + .......  = 1
27 (27) ( 27) 26
1
27

FINAL STEP Page # 60


MATHEMATICS
91. S-1 : f(0) = 0 and Lim f ( x )   .
x
Also f is continuous so f(c) = 1000, for some c  R.
S-2: This statement is false, because continuity of f is not given.

92. Use diametrical form and A(x1, y1) and B(x2, y2) as diameter.

93. First 3 equations are suggestive that


(x1, y1) (x2, y2) and (x3, y3) and the vertices of a  BCA
x y1 1
1 1
Now A = | x2 y2 1 |
2 x y3 1
3

2
x1 y1 1
 2
4A = x 2 y2 1
x3 y3 1
4A2 = 4s(s – a)(s – b)(s – c)
16A2 = 2s(2s – 2a)(2s – 2b)(2s – 2c) = RHS
2 2
x y1 1 x1 y1 1
1 1
and LHS = (16)  x 2 y2 1  4 x2 y2 1   = 4 Ans.
4 x y3 1 x3 y3 1
3

94. a(2x + 3y – 5) + b(x – 2y + 1) = 0 .........(1)


This is a family of lines passing through point of intersection of 2x + 3y – 5 = 0
and x – 2y + 1 = 0 i.e. (1, 1)
Let P(h, k) is the image of A(0, 0) on the line passing through B(1, 1).
 AB2 = PB2
 1 + 1 = (h – 1)2 + (k – 1)2
 x2 + y2 – 2x – 2y = 0 is the locus of P. Ans.

 x   x 
96. We have sin–1 x = tan 
1  > tan –1 x > tan –1 y  As x  y,  x
  
 1 x
2
  1 x2 
 S2 is true.
1 1
Also, e <  
e 
so, by S2

 1   1   1 
sin 1   tan 1    tan 1    S is also true
 e  e  x 1

Both S1 and S2 are correct and S2 is explaining S2 also. Ans.

FINAL STEP Page # 61


MATHEMATICS
97. Option (D) is correct. Statement-2 is well known fact and Statement-1 is not true.
If y  0, y4 – 8y  0

 y(y3 – 8)  0  y 2  2 y  4)  0
y(y – 2) ( 
  
Always positive

 y(y – 2)  0  y  (–, 0]  [2, )


Now, y = 0 is not possible as then yx + y2 x + 2 = 0 
2 2 = 0 (Not possible)
 Statement-1 false but statement-2 is true.

b1 b2 
a1 a2 a3  b 3 b4 
98. Let A = a a 6  , B = b
 4 a5
 5 b6 

b1a1  b 2a 4 b1a 2  b 2a 5 b1a 3  b 2a 6


So, det (BA) = b3a1  b 4a 4 b3a 2  b 4a 5 b3a 3  b 4a 6
b5a1  b 6a 4 b5a 2  b 6a 5 b 5a 3  b 6 a 6

a1 a2 a3 b1 b2 0
= a4 a5 a 6 × b3 b4 0 multiply (column by row)
0 0 0 b5 b6 0

= 0. Ans.

t12
99. = 16
t 22 y
t1 = 4t2 or – 4t2 t1
Case-1: If t1 = 4t2
h = t 1 t 2 = 4 t 22 (h, k)
k = (t1 + t2 ) = 5 t2 x

h 25
 k2 = 25 · = h
4 4 t2

25
 Locus is y2 = x
4
25 25 3
Hence, area bounded by y2 = x and x2 = 12y is 16 × × = 25.
4 16 3
Case-2: if t1 = – 4t2 y
t1
h = t 1 t 2 = – 4 t 22
k = (t1 + t2) = – 3 t2
(h, k)
h x
 k2 = 9  
 4 
9 t2
 Locus is y2 = x
4
9 9 3
Hence, area bounded by y2 = x and x2 = 12y is 16 × × = 9 Ans.
4 16 3

FINAL STEP Page # 62


MATHEMATICS
100. Let f(x) = x tan–1x
By LMVT,
f (x + 2) – f(x) = 2f '(y) for some y  (x, x + 2)
As x , we have y .
 2y 
Hence Lim [f ( x  2)  f ( x )] = Lim 2f ' ( y) = Lim  2
 2 tan 1 y = . Ans.
x  y y  1  y
 

101. C1 : zz  iz  iz  b  0
Put z = x + iy
x2 + y2 + i(x + iy) – i (x – iy) + b = 0
 x2 + y2 – 2y + b = 0 ...........(1)
Also, zz  (1  i) z  (1  i) z  4  0
 x2 + y2 + (1 – i) (x + iy) + (1 + i) (x – iy) – 4 = 0.
 x2 + y2 + 2x + 2y – 4 = 0 .........(2)
 Using condition of orthogonally, we get
2(0 × 1 + (– 1) (1)) = b – 4  – 2 = b – 4  b = 2.
 S-1 is false, but S-2 is true. Ans.

x 2 y2
102. Equation of ellipse is  1
9 8
1
Focus  (1, 0), e =
3

Also, a point on given ellipse is 3 cos , 2 2 sin  
 3 
Put  = , we get a point on given ellipse as  , 6  .
3 2 
 
Now, circle described on the focal distance as diameterof the ellipse is 2 x  3 x  1  2 y y  6  0 ,
which is true. (Using propertyof ellipse.) ]

103. Statement-1: As, incentre is equi-distant from the three sides of the triangle. Here, distances of sides
of triangle from (0, 0) is 2, so I  (0, 0).
Obviously, Statement-2 is true but not explaining Statement-1.]

104. Clearly statement-2 is wrong for example


3
if f(x) = x3 is derivable at x = 0 where f(x) = 0, then f ( x )  x is also derivable at x = 0.
2
Statement-1: f(x) = x  a x  b will be derivable for all x if a = b = 0 and vertiex x2 + ax + b will
be at origin.
 ordered pair (a, b) = (0, 0)
 number of ordered pairs = 1. Ans.

105. Statement-1 is false, statement-2 is true.

FINAL STEP Page # 63


MATHEMATICS
10 10
r  20  20  r  (20  r )! 20! 10!
10
20!
106. Let S   (1)  r  10  r    (1)
  
. r  
r!(20  r )! (10  r )!(10)! r 0
(1) r .
r!(10  r )! (10!) 2
r 0 r 0
10

20!
(10!) 2  (1)r  r 
10

20!
(10!) 2

10

C 0 10 C1 10 C 2 10 C3  ......... 10 C10 = 0 Ans.
r 0

107. Statement -1 is true and Statement -2 is false.


a b
Let A =  c d  so that
 
 a b   a b   a  bc ab  bd   1 0 
2
A2 =  c d   c d  =  2 =  
    ac  dc bc  d   0 1 
 a2 + bc = 1 = bc + d2
and (a + d) c = 0 = (a + d) b
Since A  I, A  – I
So, a=–d
1  bc b
Hence det A = c  1  bc = –1 + bc – bc = –1
 Statement-1 is true
y
But Tr · (A) = 0 and hence Statement-2 is false.
y= – 2x+5 y = 2x–5
108. f(x) = x  1  x  4  x  R
y=3
 ( x  1)  ( x  4)  2 x  5,   x  1

= ( x  1)  ( x  4)  3, 1 x  4
( x  1)  ( x  4)  2 x  5, 4x
Range of f(x) = [3, ). (0,0) x=1
x
x=4
Clearly, f ' () = 0  S - 1 is true.

But f(c) = , for some c  R is false because minimum value of f(x) is 3. Ans.
2
(AK ) AB
109. = cos  =
(OA ) AK
 (AK)2 = (AB) (OA) = (AP) (AQ) [AK2 = AP · AQ using power of point A]
AP  AQ
Also OA =
2 H
[AQ – AO = r = AO – AP  2AO = AQ + QP]
 AP  AQ  B
 (AP) (AQ) = AB   A  O
 2  P Q
2(AP) (AQ)
 AB = (AP  AQ) K
110. x1 a x2 b x3
x1 0, x2 0, x3 0 , where a and b are two selected integers and x1, x2, x3 are the number of integers
not to be selected.
Hence, x1 + x2 + x3 = 98
Total number of solution = 100C2.
Now number solutions when difference between a and b exceeds 10.
x1 + x2 + x3 = 98
Give 10 coins to x2.
Hence, x1 + x2 + x3 = 88
Then number of solution = 90C2 .
Hence, final solution is 100C2 – 90C2.
FINAL STEP Page # 64
MATHEMATICS
[MULTIPLE OBJECTIVE TYPE]
111.
dF
(a) Given 2F( x )  f ( x )  = f 2 (x) and = f (x)
dx
f 2 (x)
 F(x) =  f ( x ) ; F ' (x) = f (x) · f ' (x) + f ' (x)
2
dF
 f (x) = f ' (x) 1 f ( x )  (but  f (x ) )
dx
f (x) 1
 f ' (x) = =1 – > 0 (as f (x) > 0)
1  f (x) 1  f (x)
Hence f is strictly increasing.
f (x)
(b) Lim
x  x
 1 
or = Lim f ' ( x ) (using L'Hospital's Rule) = Lim 1   as x  ; f (x)   ;
x  x   1  f ( x ) 
f (x)
 Lim =1
x  x

112. Let the 3rd vertex be (a, a + 3)


a a 3 1
2 1 1 = 10
(a, a +3)
3 2 1 A
y=x+3

a (1  2)  (a  3) (1)  1(4  3) = 10

3a  a  3  7 = 10
B(2, 1) C(3, –2)
4a  4 = 10
4a – 4 = 10 or 4a – 4 = – 10
4a = 14 or 4a = –6
7 3
a= or a=
2 2
 7 13   3 3
A ,  or A , 
2 2   2 2

113.
    
(A) If v  w = 0  w the v  0 , otherwise the above realtion is not possible  True
rue.
      
(B) Given, a  b  3 c  0  a  b   3 c
  2  2   2
 a b =3 c  2  2 cos 3 c
3
 2 
 c  1  c  1  True.

FINAL STEP Page # 65


MATHEMATICS
     2   
If u  v  · u  v   0  u  v
2
(C) 0  u  v
 
But it does not imply that u  v .
 
e.g. Let u  î and v  ˆj
Now, î  ˆj · î  ˆj = 0
But î  ˆj  False.
 
(D) Given, a  1  b
 
Let  = a ^ b where  [0, ]
  2  2  2  
Now, 2a  b =4 a  b  4a · b = 5 + 4 (1) (1) cos  = 5 + 4 cos 

  2
 2a  b = 9, when cos  = 1 i.e.,  = 0°
max .
 
 Maximum value of 2a  b = 3. Ans.

   
(a1  a 2 ) · (b1  b 2 )
114. Shortest distance =  
| b1  b 2 |
 
where, a1 =  3î  6ˆj , a 2 = 2î  ˆj  6k̂
 
b1 =  4î  3ˆj  2k̂ , b 2 =  4î  ˆj  k̂

(5î  7ˆj  6k̂ ) · ( î  4ˆj  8k̂ )


 S.D. = = 9. Ans.
| î  4ˆj  8k̂ |
Now, verify alternatives.]

115. y
5  5
(A) if  =  = ;
6 2 12 1
/2
 1 x
O  (c,0)
now, sin = ;
2 c
c=1
1 2 2
c= 
sin 75 ( 3  1)
4
= = 6 2  (A) is not correct
6 2
 3 1 1 4
(B) = , c= = = = 5 1  (B) is correct
2 10 sin 54 cos 36 5 1
  1
(C)
2
= , c = sin( 4) =
4 2  (C) is correct

  1
(D) = , c= =2  (D) is correct
2 6 sin 30

FINAL STEP Page # 66


MATHEMATICS
116.
 2 1
 x sin 2 x  0
(A) False, f (x) = x
0 x0
 Y
(B) This statement is true. (0,1)
x
x 0  x  1 y=

For example : Let f ( x )   1 x  0 X
 0 x 1 (1,0)
(C) 2
f(x) = x – cosx + 4
Now f(–1) = 1 – (– 1) + 4 = 6
& f(2) = 4 – 1 + 4 = 7 Graph of f(x)
As f is continuous on [– 1, 2], so f (c) = 2, for some c  (– 1, 2)
(By using intermediate value theorem.)
1 1 1
(D) (f – 1)' (4) = = = .
f ' (0) 1  e x x 0 2

117.
(A) if two circles are concentric then infinite common normals  False
(B) perpendicular bisector only if two circles have equal radius  False

(C)

Now p12  r12  l 2 and p 22  r22  l 2 obviously p1 < p2  True


(D) Radical axis does not exist in case of two concentric circles  False

118. Case-1: If lines form a triangle then n = 4


i.e. 3 excircles and 1 incircle
Case-2: If lines are concurrent
or all 3 parallel then n = 0
Case-3: If two are parallel
and third cuts then n = 2
hence (A), (C), (D)

FINAL STEP Page # 67


MATHEMATICS
119. By intermediate value property
f ( 0)  f ( 2)
= f (c), 0 < c < 2
2
By mean value theorem,
f (1) – f (0) = f ' (c1), 0 < c1 < 1
f (2) – f (1) = f ' (c2), 1 < c2 < 2
By subtraction
f (0) + f (2) – 2 f (1) = f ' (c2) – f ' (c1)
= (c2 – c1) f '' (c), c1 < c < c2  f (0) + f (2) – 2 f (1) < 0
 f (0) + f (2) < 2 f (1)
1
120. Eccentricity of ellipse = .
2
x2 y2
Let equation of ellipse be   1 .......(1)
a2 2  1
a 1  
 2
x2 – y2 = 2 .......(2)
As, (1) and (2) intersect orthogonally, so
dy  dy 
   1 at point of intersection. Ans.
dx  (1) dx  ( 2)

121. Given curve is ax3 – y + b = 0  y = ax3 + b


Let point P1 be (t1, at13 + b)
dy
Slope of tangent = = 3 at12
dx P1

 Equation of tangent is y – ( at13 + b) = 3 at12 (x – t1)


 Tangent meets curve at P2 (t2 , at23 + b)
 (at23 + b) – (at13 + b) = 3at12 (t2– t1)
 a (t23 – t13) = 3at12 (t2– t1)  t22 + t12 + t2 t1 = 3 t12 (  t 1  t 2)
 t22 – t2 t1 – 2 t12 = 0  (t2 +2t1) (t2 – t1) = 0
 t2 = –2 t1
Similarly t3 = – 2t2
 abscissae are in G.P. for all values of a and b.

FINAL STEP Page # 68


MATHEMATICS
122. Three planes meet at two points it means they have infinitelymany solutions, so
2 1 1
So, 1 1 1 = 0  2 (– 3 + 1) – 1 (3 + 1) +  (1 + 1) = 0  = 4
 1 3
P1 : 2x + y + z = 1
P2 : x – y + z = 2
P3 : 4x – y + 3z = 5
P on XOY plane  (1, –1, 0) (which can be obtained byputting z = 0 in any two of the given planes.)
 1 3 
Q on YOZ plane   0, ,  (which can be obtained by putting x = 0 in any two of the given planes.)
 2 2
 Straight line perpendicular to plane P3 passing through P is –
x 1 y  1 z
 
4 1 3
1 3
PQ = î  ˆj  k̂
2 2

OP · î
Projection of PQ on x-axis  =1

 1 1 1 
Centroid of OPQ is  , , 
3 2 2
1
h p sin  h (tan h )q p 1 1
123. +
f '(0 ) = Lim h = Lim h sin  h (tan h )q
h0 h h0 h
For this limit to exist p – 1 > 0 and q  N
 p  2 and q  N
1
 h p sin  h (tan h )q
again f '(0–) = Lim h
h0 h
1
Lim h p  1 sin
 (tan h )q
h0 h
Which is same hence B, C, D. Ans.]

124. Case-I: | z | < | z – 4| ....(1)


then | z – 2 | = | z |  Re(z) = 1
which satisfyequation (1)
Case-II: | z | > | z – 4 | ....(2)
then | z – 2 | = | z – 4 |  Re(z) = 3
which satisfyequation (2)
so Re(z) can be 1 and 3 Ans.

FINAL STEP Page # 69


MATHEMATICS
125. 9  4 5  = 3 + f
n

let 9  4 5  = f '
n

I + f + f ' = 9  4 5 n + 9  4 5 n = even integer


0<f+f'<2  f+f'=1
 I is an odd integer  (A)

126.
a 
(A) Let the point be  ,   .
e 
Chord of contact is T = 0 y Q(t2)
(at 1t 2, a(t1+t 2))
a T
i.e., x   + y = a2, which is passing through (ae, 0).
e L(a, 2a)
P(t1)
(B) T (at1t2, a(t1 + t2))  at1t2 = a  t1t2 = 1
x
[Hence equation of PQ is 2x – (t1 + t2)y + 2a = 0
(x + a) – y = 0 ]
 chord PQ passes (– a, 0) which is foot of directrix L'
x=a
 (B) is also true.
(C) As | PS1 – PS2 | = constant
so, for hyperbola
0 < k2 + 1 < 5  k2 < 4  –2<k<2
So, number of integral values of k are 3 (i.e., k = – 1, 0, 1). Ans.

127. Y – y = m(X – x)
put X = 0
Y = y – mx
 y – mx = x2
dy 1
– y = – x (linear DE)
dx x
solving y = – x2 + cx
x = 1, y = 1  c=2
y = 2x – x 2 ]

î ĵ k̂
  
128. V  n1  n 2  (3î  ˆj  k̂ )  ( î  4ˆj  2k̂ ) = 3  1 1
1 4 2

= î ( 2  4)  ˆj ( 6  1)  k̂ (12  1) =  2î  7ˆj  13k̂  B and D

FINAL STEP Page # 70


MATHEMATICS
129. Given, x5 – 40x4 + x3 + x2 + x +  = 0
a a a ar ar2
r2 r

1 1 2
Now, a  2   1  r  r  = 40 ......(1)
r r 
1 2 1 1
Also,  r  r  1   2  = 10 ......(2) (Given)
a r r 
Equation (1)
 Equation (2)  a2 = 4  a = ± 2 ......(3)

As, product of roots = a5 = –  ......(4)


So, from (3) and (4), we get  = 32 or – 32. Ans.

130. (2x – 3y + z)10


10! (2 x ) (3y) (z) 
General term =  ! ! !
.......(1)
    10

12! 12 11
 Number of terms = 10! 2! = = 66.
2
Put x = y = z = 1 in (1), we get
Sum of all the coefficients 0.
Put  = 2,  = 3,  = 5 in (1),
10!  10!  4  27  10!  9
we get coefficient of x2y3z5 = (2)2 (–3)2 = =
2! · 3! · 5! 2  6  120 5!
Also, sixth term is the middle term of expansion.

131. f(x) = (x2 + 2x + 3)2 + 2(x2 + 2x + 3) + 3


x2 + 2x + 3 > 0  x  R
 f(x) > 0  x  R.
Assume x2 + 2x + 3 = y
f(x) = y2 + 2y + 3 = (y + 1)2 + 2  f(x) = (x2 + 2x + 4)2 + 2  f(x) = [(x + 1)2 + 3]2 + 2
f(x)  9 + 2 = 11  f ( x ) min
= 11. Ans.

132.
y-axis

(2,3)
(0,3)
(1,2)
(4,1)
x-axis
O(0,0)

Graph of f(x) = | x – 1| – | x – 2| + | x – 4|
Now, verify alternatives. Ans.

FINAL STEP Page # 71


MATHEMATICS
1
133. Put x = in both the quadratic equations
x

1 1
x1 x2
x2 – 4x + a = 0 ; x2 – 6x + b = 0
1 1
x3 x4

1 1 1 1
, , ,
x1 x 2 x 3 x 4 are in A.P..

1 1 1 1
Let = A, x = A + D, x = A + 2D, x = A + 3D
x1 2 3 4

1 1
  
x1 x 3 = 2A + 2D = 4 A+D=2 .......(1)

1 1
  
x 2 x 4 = 2A + 4D = 6 A + 2D = 3 ......(2)

From (1) and (2),


A = 1, D = 1
1 1 1
 x1 = 1, x2 = , x3 = and x4 =
2 3 4
1 1 1 1 1 1
 a= x x = , b= x x  8
1 3 3 2 4
Now verify alternatives.
Alternatively: We have x1, x2, x3, x4 are in H.P.
1 1 1 1
 , , , are in A.P..
x1 x 2 x 3 x 4
Let d = common difference of A.P.
x1
Now, ax2 – 4x + 1 = 0 x
3

x1  x 3 4a 1 1 1 1 
So, x x = 1 a = 4 or x  x = 4 or x   x  2d  = 4
1 3 1 3 1  1 
1
 x +d=2 .......(1)
1
Also, x2 and x4 are roots of bx2 – 6x + 1 = 0
x2  x4 6b 1 1 1   
 = = 6 or  = 6 or   d    1  3d  = 6
x 2x 4 1b x2 x4 x  x 
 1   1 
1
 x + 2d = 3 .......(2)
1

FINAL STEP Page # 72


MATHEMATICS
1
 From (1) and (2), we get  1 and d = 1.
x1

1 1 1 1
So, = 1, =2, = 3 and =4
x1 x2 x3 x4

1
Since, = a  a = 3
x1x 3

1
Also, =b  b=8
x2x4
Now, verify alternatives. ]

134. 4C · 100C + 4C 3 · 100C + 4C 2 · 100C + 4C 1 · 100C + 4C 0 · 100C


4 6 7 8 9 10

Out of 104 students of which 100 are boys and 4 are girls, we have to select 10 students.
This can be done in 104C10 = 104C94
Hence, x + y = 114 or 198 Ans.
Aliter: 100
C 6  100C7  3
  

100
C7  100C8  3   100
C8  100C9 +  100C
9 + 100C
10

= 101C7 + 3 101C
8 +3 101C
9 + 101C10
= C C 2 
101
7
101
8
101

C8  101C9  101C9  100C10

= 102C8 + 2 · 102C9 + 102C10 = 102C8 + 102C9 + 102C


9 + 102C
10
= 103C9 + 103C10 = 104C10  104C94
 x = 104 and y = 10 or x = 104 and y = 94
Hence, (x + y) = 114 or 198 Ans.

1
135. We have, = 42 3
1  sin 

 1  42 3  2 3 3  2
 1 – sin  =    =  sin  = = , ]

42 3  42 3  2 2 3 3

10 20
 2 1   1
136.  x  2  2 = x  
 x   x
1
Tr + 1 = 20Cr x20 – r (–1)rr
= 20Cr x20 – 2r (–1)r
x
 20 – 2r = 0  r = 10
So, term independent of x = 20C10. Now verify alternatives. Ans.

FINAL STEP Page # 73


MATHEMATICS
137.
n 2 1  2 n 2 3   n  4   n 2 1 
Lim 1  Lim  2  1
n  n  2 n 2  n 1  n   2 n  n 1   n 
(A) l= e =e = e2 = e
1
ln l = Ans.
2
 1  x x 
(B) Lim x    
x   e  x  1  
 
1
Let x =
t
ln (1 t )
1 1  1 
1t 1t
Lim      1 Lim (1  t )  e 1 e t e
 t 0 t  e 1  t  = e t 0 t · (1  t )1 t = 2 Lim
    e t 0 t

ln (1 t )
1
1 e t  1  ln (1  t )  1 ln (1  t )  t 1
= Lim ·  1 = Lim 2 = . Ans.
e t 0  ln (1  t )   t  e t 0 t 2e
  1
 t 
(C) As we are interested in coefficient of x50, we shall ignore all the terms with exponent more than 50.
So, (1 + x2)25 (1 + x25) (1 + x40) (1 + x45) (1 + x47)
= (1 + 25C1 x2 + ............... + 25C25x50) × (1 + x25 + x40 + x45 + x47) = 25C25 + 25C5 = 1 + 25C5.
(D) (A–1B–1)T = (B–1)T (A–1)T = (BT)–1 (AT)–1 = B–1 A–1 = (AB)–1 = (BA)–1 = A–1 B–1
 A–1 B–1 is symmetric matrix. Ans.

138. tp + 1 = nCp xp in (1 + x)n


Here p + 1 = (4r – 4)
p = 4r – 5
also, p + 1 = r + 14  p = r + 13
 28C 28
4r – 5 = Cr + 13
either 4r – 5 = r + 13  r = 6 Ans.
or 4r – 5 + r + 13 = 28
5r = 20  r = 4. Ans.

Q.139 log3 | 2z + 1 | < log3 |2z – 1| (as cot–1x is a decreasing function)


 |2z + 1 | < | 2z – 1|
or (2z + 1) ( 2 z  1) < (2z – 1) ( 2 z  1)
2 (z  z ) < – 2 (z  z )
zz <0
2Re (z) < 0
 Re(z) < 0

100
Q.140 Clearly,  100 Ck (x  2)100k 3k = (x  2)  3100 = (1 + x)100
k 0
Hence, coefficient of x50 is 100C50. Now verify
Option (A), (D) equal to 100C50.
(B) 99! (100C50)
(C) 100 C .
49
FINAL STEP Page # 74
MATHEMATICS
141.
n
(A)  [r ·2r · 31  r ·2r · 32.......  r ·2r · 3n ]
1 2 n
r 1
6(6n  1)
6+ 62 63
+ + ..... +  S=
6n = 1554
5
or 6(6n – 1) = 7770 or 6n – 1 = 1295  6n = 1296  n = 4 Ans.
y

y = |x – 2| 2
//////
///
/ ///
/// ////
///
///
r=2
/// ///
///
(B)
///
/ /// ///
///
/// ///
// /// ///
/ ///
///
y = |x – 2| – 2 ///
///
/ ///
/ ///
///
/
x
///
/// /

O 1 2

(C) Domain : x2 – 7x + 10 < 0  (x – 5) (x – 2) < 0  x  (2, 5)

Also, cos   x 2  7  = 1   x 2  7 = 2n, n  I  x 2  7 = 2n


 
Only value of x = 3  Sum = 3. Ans.

142.
(A) Clearly smaller circles passes through the centre of the larger circle
y
AB = longest chord = 52 A
and PQ is the smallest chord P

PQ = 2 (26) 2  (10) 2  PQ = 48 26

 Difference = 4 Ans. C2
(B) f(x) = 1 + Cx C1
x
(5,0)
1 1 1
C1
 f (t ) dt =  (Ct  1) dt = 2  (Ct  1) dt
2 2 2
where C =
1 1 0
Q
1
 Ct  3
C  C B
C=2   t  = 2  1  = 2  C = 6. Ans.
 3 0 3  3

(C) cos3x = 1 – (sin2x)3 = (1 – sin2x) (1 + sin4x + sin2x)


 3
cos2x = 0  x = or x =
2 2
4 2
cos x = 1 + sin x + sin x
(1 – cos x) + sin2x (1 + sin4x) = 0
(1 – cos x) + (1 – cos x) (1 + cos x) (1 + sin4x) = 0
cos x = 1  x = 0
1  (1  cos x ) (1  sin 4 x ) = 0
 
always positive

 3
Hence, x = 0 or or  3 solution. Ans.
2 2
FINAL STEP Page # 75
MATHEMATICS
143.
3
x2 2
(A) e > 0 for all x hence  e x dx > 0 or < 0 (think!)

3
2
Hence,  e x dx = 0 is possible only if  = 3

 = 0, 1, – 1
 3 values. Ans.
(B) 2a – 3b + 18 = 0 .......(1)
 b 1   2  (3, – 1)
   =–1
 a 3  5
(a, b) 2x – 5y + 18 = 0
2b + 2 = 15 – 5a
5a + 2b = 13 .......(2)
Solving (1) and (2) a = 1 and b = 4
Hence, (a + b) = 5, which is twin prime with 3 and 7.
10 
(C) x2 – x+c=0
9 2
10
 + 2 =
9
9 + 9 – 10 = 0  (3 + 5) (3 – 2) = 0
2

2 5
= or  =
3 3
2 4 10  2 
If  =     +c=0
3 9 9 3
8 m
c=   (m + n) = 35. Ans.
27 n
5
If  = , then c < 0 hence rejected.
3
(D) Case-I:
If 0 < a < 1 (obviously 'a' can not be < 0)
then for f (x) to be increasing
3 
4ax – x2 should be decreasing in  , 2 
2 
3
  2a and 2 < 4a
2
3 1  1 3
 a and a>  a  , 
4 2  2 4
Caser-II: If a > 1 then for f (x) to be increasing
3 
4ax – x2 increasing in  , 2 
2 

FINAL STEP Page # 76


MATHEMATICS
 2a  2  a  1 but a  1;  a>1
 1 3
 final answer is  ,   (1,  ) . Ans.
 2 4

144.
(A) Let z = x + iy, i = 1
3i 3
 (x + iy)2 = 1 –  x2 – y2 = 1 and 2xy =
4 4
9 25
Now, (x2 + y2)2 = (x2 – y2)2 + 4x2y2 = 1 + =
16 16
5
x2 + y2 =
4
also, 2 2
x –y =1
—————
9 3 3
2x2 =  x= or
4 2 2 2 2
3  2
If x = then y = .
2 2 4
3 2
If x = then y =
2 2 4
3  2  3
 (Re z) (Im z) =  = . Ans.
2 2  4  8
(B) n(S) = 63 = 216
Let a, b and c are the throws on 1st, 2nd and 3rd roll, hence we must have a  b  c
Let b – a = x1 and
c – b = x2
add, ————
c – a = x1 + x2
c = x1 + x2 + a  6 (As, a  6)
but a  1, giving a = 1, we have
x1 + x2 + a  5.
To solve this inequality we introduce a false beggar say T
x1 + x2 + a + T = 5
 O O O O O ØØ Ø
n(A) = 8C3 = 56
6 7
p= =  p = 7. Ans.
216 27

FINAL STEP Page # 77


MATHEMATICS
Aliternatively:
abc
we have 4 cases
(i) a < b < c  6C3 · 1 = 20
(ii) a = b = c  6C 1 = 6
(iii) a = b < c  6C2 = 15
(iv) a < b = c  6C2 = 15
———
n(A) = 56. Ans.
(C) Domain x > 0
for x > 1, x2 > 1
x2 + x > x + 1
2
x
 2x  2 x 1 and log2 x > 0
2
x
 2x  log 2 x  2 x 1 for all x > 1
Again for 0 < x < 1
for x < 1
x2 < x
x2 + x < x + 1
2
x
2x  2 x 1 and log2 x is – ve
2
x
 2x  log 2 x  2 x 1
for all 0 < x < 1
 x = 1 satisfies Ans.
145.
(A) s = 7C2 – 3 · 3C2 + 3 (think!) G H
s = 21 – 9 + 3
s = 15 G
F C
t = 7C2 – 3 (think!)
t = 32
Now, (s, t)  (15, 32)  (A)] E D
(B) Since, lines are perpendicular  1–p+q=0 .....(1)

1 p 1
and coplanar  1  1 q = 0 2pq – 2p – q + 1 = 0  (2p – 1) (q – 1) = 0
2 1 2

1
 p= or q = 1 .....(2)
2
1 1
So, when p = ,q= or q = 1, p = 2. Ans.
2 2

FINAL STEP Page # 78


MATHEMATICS
 
 a b  7,  4,  4  2,  1, 2 
v =        v =  
 
(C)
| a | | b |  
   9 3 

 
 v =
9

î  7ˆj  2k̂ 
 3 6
|v| = 6 =  = 3
9
 
 v · a = 9. Ans.

146.
(A) A(0, 0), B(t), C(–t)
BC = 12t, y2 = 4ax, a = 3

BD 1 2 1
D is mid-point of BC and = tan 30° =   .
AD 3 t 3

 BC = 4at = 24 3 . Ans.

(B) D is mid-point of chord BC ; BAD = 45°


AB = ?
2
B (at2, 2at) ; = tan 45°  t = 2.
t
Now, (AB)2 = 4a2t2 + a2t4
= 9(4 × 4 + 16) = 9 × 32
 AB = 12 2 . Ans.

(C) Normal at P(t1) cuts curve at Q(t2) ; S(a, 0)


2
Also, t2 = – t,
t
Now, PS  QS

2at 2 2at 2
 a 
t12 1  a t

2
2
  1
1  – 4 t1t2 = (t12 – 1) (t22 – 1)

2
 1
 4  t1   = (t12 + 1)2  t1 = ± 2
 t1 

   
2
1
 (PQ)2 = 9(5)2 (4 + 1) = a2 (t1 – t2 )2 × 4   t1     PQ = 15 5 . Ans.
  t1  
 

FINAL STEP Page # 79


MATHEMATICS

(D) If PQ is a normal chord, P(t1), Q(t2), QOP =
2

2
then t1t2 = – 4 and t2 =  t1   t 12 = 2  t 1 = 2
t2

 t2 = –2 2

 
(PQ)2 = a2 (t1 – t2)2 4  t1  t 2 2 
 
9 3 2  4 
2


 2   = 81 × 2 × 6
2

 PQ = 18 3 . Ans.

147.
y y
(C) Let = m'. So on putting y = m'x in the given circle, we get (x,y)
x
(x – 3)2 + (m'x – 3)2 = 6  x2(1 + m'2) – 6x(1 + m') + 12 = 0
(3,3)
Putting discriminant = 0 (x,y)
 m' = 3 ± 2 2
So m = 3 – 2 2 and M = 3 + 2 2 x
O
Hence (M + m) = 6 Ans. (0,0)

(D) Clearly sum of radii Y


   
= + + +
s sa sb sc
(0,3)
 1 435 
 As   (4)(3)  6 and s   6
 4 2 
X X
6 6 6 6 O (4,0)
= + + + = 1 + 2 + 3 + 6 = 12 Ans. (0,0)
6 63 64 65 3x
+4
y=
12
(E) equation of the circle touching the line 2x + y + 2 = 0 at (–1, 1) is Y
(x + 1) + (y – 1) + (3x + y + 2) = 0
2 2

it passes through (3, 5)


 16 + 16 + (9 + 5 + 2) = 0  =–2
 equation of circle is
(x + 1)2 + (y – 1)2 – 2(3x + y + 2) = 0
x2 + y2 – 4x – 4y – 2 = 0  centre (2, 2)  Sum = 4. Ans.

FINAL STEP Page # 80


MATHEMATICS
148.
x 1 1
2
(A) Let f(x) = sin (2x ) 2 x 1
x3 3x 4 1
Now, f(– x) = – f(x) = g(x)
2
1 1 1
 f(x) · g(x) = –  f ( x )  2 or f(1) · g(1) = –  f (1)  = – 0 2 1 = – (–2)2 = – 4. Ans.
2

1 3 1
(B) Since the equations are consistent, so put  = 0
(a  1)3 ( a  2) 3  (a  3)3
 (a  1) ( a  2) (a  3) = 0
1 1 1
Put x = a + 1, v = a + 2, w = a + 3  u – v = – 1, v – w = – 1, w – u = 2
Also, (u + v + w) = 3a + 6
u3 v3 w3
 u v w = 0  (u – v) (v – w) (w – u) (u + v + w) = 0
1 1 1
 (– 1) (– 1) (2) (3a + 6) = 0
 a = – 2  | a | = | – 2 | = 2. Ans.

 1 1
 (1  3t  2t 2 ) t  (1  3t  2 t 2 ) t   0 
(C) Let L = Lim    
t0 

t   0 
 

 1 1 
 (1  3t  2t 2 ) t  e3 (1  3t  2 t 2 ) t  e3 
= Lim   
t0  t t 

 
 L = L1 – L2 ........(1)
5 3  13 3
Now, L1 = e and L2 = e
3 2
 5 3 13 3 8 3
 L= e  e = e = 4e3  ke3  k = 4 Ans.
2 2 2
35 35 
6 6 
(D) Let I =  3 sin x  4 cos x dx = 5

 sin( x  ) dx = (5  6)  sin x dx = 60.
 0
6 6

Hence 60 is divisible by 2, 4, 6, 10 Ans.

FINAL STEP Page # 81


MATHEMATICS
149.
(A) We have f(x) = ax3 – 9x2 + 9x + 3
 f '(x) = 3ax2 – 18x + 9 = 3(ax2 – 6x + 3)
As f(x) is strictly increasing on R, so
a > 0 and Disc  0  36 – 12 a  0  a  3
So, a  [3, )
Hence, number of integral values of a, a  [ – 5, 100] are 98. Ans.
(B) Put cos x = t. So, t  [– 1, 1]  x  R
Let g(t) = t3 – 6t2 + 11t – 6 = (t – 1) (t – 2) (t – 3), t  [– 1, 1]

g(t)

+
t=2
t
t= –1 O (0,0) t=1 t=3

(0,–6)

(–1,–24)

–

Clearly, range of g(t) for t  [– 1, 1] is equal to [– 24, 0]. Ans.


(C) We have, x3 – y2 = 0
Differentiate both sides with respect to x, we get
dy dy  3x 2 3 16m 4
3x2 – 2y =0 = = =3m
dx dx  P ( 4 m2 , 8m3 ) 2y 16m3
Let Q be (4m12, 8m13)
1
 Slope of normal at Q =
3m1
As, tangent at P is normal at Q, so
1 1
3m = 3m  m1 = – .........(1)
1 9m
Also, slope of PQ
3
8(m 3  m1 ) 2(m 2  m12  mm1 )
= = = 3m
2
4(m 2  m1 ) m  m1
 2m2 + 2m12 + 2mm1 = 3m2 + 3mm1  2m12 – mm1 – m2 = 0
 2m1 – 2mm1 + mm1 – m = 0  2m1 (m1 – m) + m (m1 – m) = 0
2 2

 (2m1 + m) (m1 – m) = 0

FINAL STEP Page # 82


MATHEMATICS
but m1  m,
 1   1 
so, 2m1 + m = 0  2 m =0  As, m1  
 9m   9m 

2 2 4 4 4
 = m  m2 =  4 = 2 = = 81. Ans.]
9m 9 m 2 4
  81
9
(D) (sin  – cos ) (tan  + cot ) = 2
sin   cos 
 =2  sin  – cos  = 2 sin  cos 
sin  cos 
let y = sin  – cos   y2 = 1 – 2 sin  cos 
1 y2
 sin  cos  =
2
 1 5
 y = 1 – y2  y2 + y – 1 = 0  y=
2
 5 1
 y = sin  – cos  =  2 
 

(cos 2   sin 2 ) (sin   cos ) 2


(sin  + cos ) (tan  – cot ) = (sin  + cos ) = ·y
sin  cos  sin  cos 

(1  2 sin  cos ) y (1  1  y 2 ) y 2( 2  y 2 ) y 2(1  y) y


= = = 2 =
sin  cos  1 y 2 (1  y ) y
 
 2 
 

 5 1 
= 2(1 + y) = 2 1   =
 5  1]
 2 
150. B' C ' = 7 = a'
A' B' = 12 2  9 2 = 144  81 = 15 = c'

A’ C’ = 12 2  162 = 144  256 = 20 = b'


where A'B'C' is the pedal triangle of ABC note that altitudes bisects, each interior angle of the pedal
triangle ie  – 2A,  – 2B and  – 2C
O  x , y  is the orthocentre of  A BC & also the incentre of  A ' B ' C '

ax1' bx '2 cx 3' where a', b', c' are the sides of the pedal triangle
x
a '  b '  c'
7  20  15  8  20  8

15  7  20
70  60  80 70  60  80
 
15  7  20 21
210
  10
21
FINAL STEP Page # 83
MATHEMATICS
a ' y'1  b' y'2  c' y'3
7  25  15  9  20  16
y  = 15
a '  b '  c' 42
 orthocentre 'O' ( 10 ,15) Ans.(i)
25  15
Now slope of OA’ = 1  slope of BC = –1
20  10
16  15 1
slope ofOB’ =   slope of AC = 2
8  10 2
9  15 1
slope of OC ‘ = 3  slope of AB = 
8  10 3
1
 equation of AB is y – 9 =  ( x  8)  3y + x = 35 ....(i)
3
equation of AC is y –16 = 2 ( x – 8)  y + 2x = 0 ....(ii)
equation of BC is y –25 = –1 ( x – 20)  y + x = 45 ....(iii) Ans.(ii)
Solving we get co-ordinates of
A (5 ,10 ); B( 50 , –5 ); C( 15 , 30 )
 70 35 
centroid of the triangleABC is  ,   (a, b)
 3 3
Hence a + b = 35 Ans.(iii)
 70 35  2 1
Now, orthocentre 'O' ( 10 ,15) and centroid  ,  (10,15) G (a,b)
 3 3
 70 35 
70 (1)(10)  15a  , 
=  15a = 60  a = 4  3 3
3 3
35 (1)(15)  2b
=  2b = 20  b = 10
3 3
Hence circumcentre of the triangle ABC is (4, 10) Ans.(iv)

FINAL STEP Page # 84


MATHEMATICS

2 2 2
151. We have, x  (7  k ) x  7k  ( x  3)( x  3k  2) = 0

 ( x  7)( x  k 2 )  ( x  3) x  (3k  2)  = 0
The above equation has a solution if equations (x – 7) (x – k2) =0 and (x – 3) (x – (3k – 2)) = 0
has at least one common root.
 k2 =3  k = ± 3
or k2 = 3k –2  k2 – 3k + 2 = 0
 (k – 1) (k – 2) = 0  k = 1 or 2
3k –2 = 7 k = 3
 Sum of all values of k is = 3 – 3 + 1 + 2 + 3 = 6 Ans.

152. Given r0 = 6 A

1
sin 30° =
2
C0
6  r1
= 6r
1
r 0 – r1
r1 = 2 C1
2
|||ly r2 = 30° r r1
3 30° 1
Hence, O B
2 6
S=6+2+ + .........= 1 = 9. Ans.
3 1
3

  13
153. sin   tan 1 x  =
6  14
14 13
  13  1  13 
+ tan–1x = sin 1  = tan  
6  14  3 3 3 3

 13 1 
 
 13  1  1  1 3 3 3  tan 1  10  9 
tan–1x = tan 1   tan   = tan   =  3 3  22 
3 3  3  1  13    
 9 

5 3 
tan–1 x = tan 1  
 11 

5 3 a 3
 x=   a + b = 16. Ans.
11 b

FINAL STEP Page # 85


MATHEMATICS

a  2 
154. Let

= = k2 .......(1)   ah  a ,  a  h 
 ' a '2   ' a ' h ' a '2 a ' h ' 

s abc
Now = a '  b'  c' = k ......(2)
s'
 r' r' k ' r' 1 1
 · =k  =  =k 2 =
r ' r  r k k
r' a' h' h  2r 1
 = = = = ah1
r a h h 2 A
b' c' h'
1 B1 C1
r' 2r 2 2 · ah a'
h
=1– =1– =1– 2
r h hs hs
B a C
r' a
 1
r s
ra a
=1–
r s
rb b
|||ly =1–
r s
rc c
=1–
r s
ra rb rc abc
 on adding + + =3– =1
r r r s
s(s  a )(s  b)(s  c) 11· 6 · 3 · 2 6 11
  ra =r=
s
=
11
=
11

 n 1 n 1 
n  2n  2  2n  2  n   
2 2 
155. Sum =  tan 
1
 =  tan 1

 n  1 n 1 
(divide by 2)
n 1  2  n 2 1  n 1
1  2
·
2 

n  n 1 n 1   n 1  n 1 1
=   tan 1 2
 tan 1
2 
 = tan 1 
  2 
  tan 1
 2
 tan 1
2
n 1  

n
 1  
Lim sum    tan 1 
 2 
 1  
 =  cot 1 
2  2
 =  tan 1 2 .
 2
 
 k = 2  k2 = 2. Ans.

FINAL STEP Page # 86


MATHEMATICS
157. For consistency,
a 1 b
b 1 a =0
a b ab
 (a – b) (a – b2) = 0
 Either a = b or a = b2
When a = b, then ordered pairs are (0, 0), (1, 1), (2, 2), (3, 3), (4, 4).
When a = b2, then (4, 2)
Hence, number of ordered pairs are 6. Ans.]

 
2 2
 1 x  1 x  1 x  2 2
158. f(x) =  cos    sin   sin   x  6x  8
 2 2  2 12

=
 1 x


cos
2
 sin 1 x  

2
 cos 1 x
2
 sin 1 x 
2
   sin 1 x

2 12
2 2
x  6x  8  

=

2
x  x x 2 2
cos 1  sin 1   sin 1 
2 2 2 2 12
x  6x  8  
 
2
=
  1 x
2
 cos
2
 sin 1 x  
 
2  12
x 2
 6 x  8 =
 2 2 2

4 12
x  6x  8  
Domain of f(x) is [– 2, 2]
f(x) is increasing in [– 2, 2]
  2 9 2  1 9
 Range of f(x) is  4 , 4  = [a2, b2]  2(a + b) = 2    = 5. Ans.
  4 4

159. log sin 2 x (2)  log cos 2 x (2)  2 log sin 2 x (2) · log cos2 x (2)  0
 
(a ) (b)

a + b + 2ab = 0 (say)
1 1
  2 0
a b
 log 2 (sin 2 x )  log 2 (cos 2 x )  2  0
1
 log 2 (sin 2 x · cos 2 x )  2  0  sin2x · cos2x = 2–2 =
4
1 
 sin2x · cos2x =  sin22x = 1  sin2x = sin2
4 4
 n   
 2x = n ±  x=  = (2n ± 1) = (4p – 1) , p  I
2 2 4 4 4
 x  (– , 2)
 3 5 7 
x =
, , ,
4 4 4 4
Total number of values = 6 Ans.

FINAL STEP Page # 87


MATHEMATICS
160. Given 4 tan x (1 + tan2 x) – 3 (1 + tan2 x) = 0
 (1 + tan2 x) (4 tan x – 3) = 0
3
 tan x = >0
4
  3 4
Hence x   0,   sin x = and cos =
 2 5 5

3
x sin x 1 x 1 x 3
 tan = = 5 =  sin = and cos =
2 1  cos x 1  4 3 2 10 2 10
5

1
x
x sin
13
2 1
 tan = = =
4 x 3 3  10
1  cos 1
2 10

x x
 cot = 3  10  cot2 = 19 + 6 10  E = 19 Ans.
4 4

A 2A
161. We have, 2 cos cos B  2 cos cos C = 2 sin A
3 3

 A  A  2A   2A 
cos  B    cos  B    cos  C    cos  C   = 2 sin A
 3  3  3   3 

 A  A
 BC   BC 
 A  B  C  3  + 2 cos  B  C  A  3  = 2 sin A
2 cos   cos   cos 
 2   2   2   2 
    
Vanishes

 A
 BC 
BCA 3  = sin A
cos   cos 
 2   2 
 

 A
   2A   BC 
 cos   cos  3  = sin A
 2   2 
 

 A
 BC 
sin A cos  3  = sin A
 2 
 

FINAL STEP Page # 88


MATHEMATICS
As sin A  0

 A
 BC 
 cos  3  =1
 2 
 

A A 2A
 B–C+ =0  + – A – C – C=0  = + 2C
3 3 3
3
3 = 2A + 6C  A + 3C =  (m + n) = 5
2
A
If B – C + = 4 or higher angle then A + 3C is negative.
3
A 15
If B – C + = – 4 then A + 3C = which is not possible. Ans.
3 2

162. 3x2 – 2x – 1 < 0


3x2 + 3x – x – 1 < 0
–1 1
(x + 1) (3x – 1) < 0 3
 1
 (3a – 2 – a2) x + 2 < 0 for some x in   1, 
 3

     1
a 2  3a  2 x  2 > 0 for some x in   1, 
  3
f (x)

Note : f(–1) = – a2 + 3a – 4 < 0. –1 1


3
1
 f    0 for solution.
3
 a2 – 3a – 4 > 0
(a – 4) (a + 1) > 0
 a  (–, –1)  (4, )  Minimum natural a is 5. Ans.

163. A3 + 3A2B + 3AB2 + B3 = (A + B)3


 AB = BA
 4 a  1  2  4  a  8  4a 
AB =  =
 1 b  1 4   1  b 2  4b 

1  2  4 a  6 a  2b 
BA = 1 4   1 b  = 0 a  4b 
     
AB = BA
 4+a=6  a=2
 –1+b=0  b=1
6 0
AB = 0 6 = 6I
 
Now, ABB–1 = 6B–1  A = 6B–1  A–1AB = 6A–1  B = 6A–1
 (A + B) = 6 (A–1 + B–1)   = 6. Ans.

FINAL STEP Page # 89


MATHEMATICS
1 1
164.  log 3   = k, k  I; log 3   = k  x = 3–k
x x
possible values of k are – 1, 0, 1, 2, 3, ........
1 1 1  (1 3) 1 9
S = (3 + 1) +   2  3  .......  = 4 + 1  (1 3) = 4 + = Ans.
3 3 3  2 2

n1
5 1
165. The probability that one rolls one's die for n times until 6 appears is =   ·
6 6
Let P denotes the chance that number of times 'A' rolls his die is equal to or within one of the number of
times 'B' rolls his die, then (P is not possible).
P = P1 (P1 + P2) + P2 (P1 + P2 + P3) + P3 (P2 + P3 + P4) + .......
2 3
1 5 1 5 1 5 1
P1 = ; P2 = · ; P3 =   · ; P4 =   ·
6 6 6 6 6 6 6

 
 P = P12  P22  P32  P42  .......  2 P1P2  P2P3  P3P4  .......

2 2 2
1 5 1 5 1
4 2  1  2 5  5 3  1  2  5 5  1  2 
P =              .......  2   ·            ......
6 6 6 6 6  6  6  6   6   6   6  

2 2
1 1 5 1  10 
  2  · 1
 6  6 6 6 2  6  16  1  8 m
P=  = 2 2 =   = =
5
2
5
2 6 5 6  11  33 n
1   1   2
6 6 6

 m + n = 41. Ans.

166. Let the time taken to solve the first problem be t and r is the common ratio, then
tr + tr2 + tr3 + .......... + trn–1 = 63.5 .......(1)
2 3 n–2
t + tr + tr + tr + .......... + tr = 127 .......(2)
2 3 n–1
tr + tr + .......... + tr = 31.5 .......(3)

(1) tr (1  r n 1 ) 1 1
= n 1 =  r=
( 2) t (1  r ) 2 2
(1) – (3)  tr = 32  t = 64
Now from (1)

tr (1  r n 1 )
= 63.5  2 × 32 (1 – rn–1) = 63.5  64 – 63.5 = 64rn–1
1
1
2

1
 = rn–1  2n–1 = 27  n = 8. Ans.
128

FINAL STEP Page # 90


MATHEMATICS
 
167. tan (7 cos x) = tan   7  sin x 
2 

 7 cos x = n + – 7 sin x
2
 1
7 (cos x + sin x) =  n   
 2
2n  1   2n  1 2n  1
sin x + cos x =  sin   x = =
14 4  14 2 19 · 796

 n  0, 1, 2, 3, 4, 5, 6, 7, 8, 9 or  10,  9,  8, ........,  1 .
 
20 solutions 20 solutions

 Total = 40 Solutions. Ans.

168. We have

4 9 16  4 9 16 
    (1) =     (x + y + z)
z 
As, x  y  z  1 (given)
x y z  x y

 4y 9 x   9z 16 y   4z 16 x 
= (4 + 9 + 16) +    +    +     29 + 2 36 + 2 144 + 2 64
 x y   y z   x z 

= 29 + 12 + 24 + 16 = 81
Aliter: x + y + z = 1 (Given)
Let x = cos2 , y + z = sin2
x = cos2 , y = sin2  cos2, z = sin2 sin2 
4 9 16 4 9 16
   = cos 2   sin 2  cos 2   sin 2  sin 2 
x y z
= 4 (1 + tan2 ) + 9 (1 + cot2) (1 + tan2 ) + 16 (1 + cot2) (1 + cot2 )
    
= 4  4 tan 2  + 9 1  cot 2  tan 2  cot 2  tan 2  + 16 1  cot 2  cot 2  cot 2  cot 2  
= 29 + (4 tan2  + 25 cot2 ) + (9 tan2  + 16 cot2 ) + 9 cot2  tan2  + 16 cot2  cot2 
= 29 + ( 4 tan2  + 25 cot2 ) + (9 tan2  + 16 cot2 ) (1 + cot2 )
 29 + (4 tan2  + 25 cot2 ) + 24 (1 + cot2 )
 53 + (4 tan2  + 49 cot2 )
 53 + 2(2 × 7) = 53 + 28
 81. Ans.

169. The different ways of getting exactly 3 consecutive sixes in 6 tosses, when a die is thrown
666 × × or  non six throw ; ×  any thing from 1 to 6
666 × or
× 666 or
×× 666
3 3 3 3
The probability are                       
1 5 5 1 5 5 1 5 5 1
6 6 66 6 66 6 66
3
1 5  5 5  5  22  55 k
=     1    1 =  =   k = 55. Ans.
 6   6   6 6  1296  6  3888 3888
FINAL STEP Page # 91
MATHEMATICS
170. We have + (2k + 1)x + (k2 + 2) = 0
2
x

 2
 Sum of roots =  + 2 = – (2k + 1)

 ( 2k  1)
 = ……(1)
3
Also, product of roots = (2) = (k2 + 2)
 22 = k2 + 2 ……(2)
 On putting the value of from  equation (1) in equation (2), we get

2
(4k2 + 4k + 1) = k2 + 2
9
 k2 – 8k + 16 = 0  (k – 4)2 = 0
 k=4
So, roots are –3, – 6. Ans.

100
 k  100
100  k  1  1   100 100 
C k =   Ck  
100 100
Ck
171. Let S     Ck =  k  1
100
k  0   k 1
k  0  k 1 k0   k0

100
1  01 100 

100
100
= 2  101  C k  = 2100  1
k0  k 1  101
 101Ck 1
k0

100  2101  1  (101)2100  2101  1 99 (2100 )  1 a (2100 )  b


= 2   101  = =  (Given)
  101 101 c
So, a = 99, b = 1, c = 101
Hence, (a + b + c)least = 99 + 1 + 101 = 201. Ans.

172. Given sum


 2    2 2   2 4  
= 3 +  cot  1 +  cot  1 +  cot  1
 7   7   7 

 2  2 4 
 1  tan 1  tan 2 1  tan 2 
7  7  7 
=3+2 
 2 tan 2  2 tan 2
2
2 tan 2
4 

 7 7 7 

  2 2 4 4 
= 3 + 2  cot · cot  cot · cot  cot · cot 
 7 7 7 7 7 7
= 5 [As, in ABC, cot A · cot B = 1] . Ans.

FINAL STEP Page # 92


MATHEMATICS
173. – 2  sin x – 3 cos x  2
4m  6
hence – 2  2
4m
2m  3
–1 1
4m
2m  3
nowif 1
4m
(2m  3)  (4  m)
0
4m
3m  7
0
m4
2m  3 ( m  4)  ( 2m  3) ( 2m  3)  ( m  4)
Again – 1  or 0 or 0
4m 4m m4
m 1
0
m4
 7
hence m   1,   m  {–1, 0, 1, 2}
 3

174. Given,
a + 19d = log1020 .......(1)
a + 31d = log1032 .......(2)
(2) – (1)
32
12d = log10 = log1016 – 1
20
12d = 4log102 – 1
12d  1
log102 = .......(A)
4
Again (2) + (1)
2a + 50d = log10640 = 6log102 + 1
2a  50d  1
log102 = .......(B)
6
12d  1 2a  50d  1
 =  36d + 3 = 4a + 100d – 2
4 6
4a + 64d = 5
5
a d = 4
16
th
17 term

5 p
Hence, 17th term is rational and its value is =  (p + q) = 9. Ans.
4 q

FINAL STEP Page # 93


MATHEMATICS
175. All 8 tails T T T T T T T T H H  1 way
Exactly 7 tails T T T T T T T H H  7C1 = 7 ways
7 gaps and one to be selected for a head
Exactly 6 Tails T T T T T T H H  6C2 = 15 ways
6 gaps and 2 to be selected for heads
Exactly 5 Tails T T T T T H H  5C3 = 10 ways
5 gaps and 3 to be selected for heads
Exactly 4 Tails T T T T H H  4C4 = 1 ways
Total ways = 34
34 17 p
 Required probability =   p = 17. Ans.
1024 512 512

176. D E E P M A LA
The condition in the problem indicates
that the letters D E LE must appear in that order.
Into this sequence of letters there are 2 position into which M can be placed on D E L × E ×
[× for filling M].
Say D E L E M or D E L M E
Now remaining three letters areAA P.

8 3!
Now they can be place in any 3 places out of 8 into C3 · ways and 5 letters
2!
D E L E M or D E L M E in order on the remaining five places.
3!
Hence total ways 2 · 8C3 · = 56 · 6 = 336. Ans.
2!

1 1 3
177. = ab sin C  3 = 2 b sin C  = sin C
2 2 b

2 [cos C + 3 sin C] = b + c .......(1)

a 2  b2  c2 4  b2  c2
cos C = =
2ab 4b

 4  b 2  c2   
 2   2 3  3  = b + c
4b   b 
   

4  b2  c2 6
  = b + c  4 + b2 – c2 + 12 = 2b2 + 2bc
2b b
 b2 + c2 + 2bc = 16  (b + c) = 4. Ans.

FINAL STEP Page # 94


MATHEMATICS
178. Obviously, all the 3 throw's can't be 2.
Case-I: Let two of dices show up the face 2.

2 2 4 P=
216
Case-II: Exactly one dice rolls 2
1 1 2 ; 1 2 3
2 1 3 ; 2 3 5
3 2 5 ; 2 4 6
7
4 2 6 P=
216
8 1
Total P= =  k = 27.
216 27
Sum = 9. Ans.

179. This problem can be stated equivalently as there are counters in 4 different colours and there are atleast
3 of each colour. Counters are all alike except for the colour. Number of ways in which an arrangement
of six counters can be made if each arrangement consists of counters of all four different colour.
Now, A B C D A ××××××
two of some
other colour
When exactly one colour repeats say B ××××××
A B C D AA C ××××××
D ××××××
6!
then, number of ways = 4C1 · = 4 (120) = 480
3!
When exactly two colours repeat say A B C D A B
 6! 
then, number of ways = 4C2   = 180 · 6 = 1080
 2! 2! 
Hence, total = 480 + 1080 = 1560. Ans.

180. We have
sin x·cos x sin 3x·cos 3x sin 9 x·cos 9 x
Q= + +
cos 3x · cos x cos 9 x · cos 3x cos 27 x · cos 9 x

1  sin(3x  x ) sin(9x  3x ) sin(27 x  9x ) 


  

2  cos 3x · cos x cos 9x · cos 3x cos 27 x · cos 9x 

1
 (tan 3x  tan 4x )  (tan 9 x  tan 3x )  tan 27 x  tan 9x  = 1 (tan 27x – tan x)
2 2
1
 Q= P  P = 2Q
2
Hence,  = 2. Ans.

FINAL STEP Page # 95


MATHEMATICS

 k  2  2k  3  2
181.   sin x    cosec x  2
k 3
2 k 
   
t 1
t

1
t =2  t=1
t
k 2k  3
 sin2x = 1  sin2x =
2k  3 k
2k  3
0< 1
k

3 
 k   , 3
2 
 integral values of k are 2 & 3
for k = 2
1  3 5 7
2sin2x = 1  sinx = ±  x= , , ,
2 4 4 4 4
for k = 3
 3
sin2x = 1  sinx = ± 1  x = ,
2 2
 Number of values of x in [0, 2] are 6.

182. We have (3 sin x – 4 cos x)2 – (b2 + b + 5) | 3 sin x – 4 cos x | + (b3 + 3b2 + 2b + 6) = 0 ....(1)
Put 3 sin x  4 cos x = t
so, the equation becomes
t2 – (b2 + b + 5) t + b2 (b + 3) + 2(b + 3) = 0
2 2
or t – (b + b + 5) t + (b2 + 2) (b + 3) = 0

t = b2 + 2 t=b+3

As t = 3 sin x  4 cos x  [0, 5] x  R


 Either
0  b2 + 2  5 or 0b+35
or b2  3 or –3b2
  3b 3
 Possible integral values of b are
– 3, – 2, – 1, 0, 1, 2
Hence, the number of integral values of b are 6. Ans.

FINAL STEP Page # 96


MATHEMATICS
 
183. Put tan–1 x = t, where t   , 
 2 2   f(t)
2 0 2
2 t
So, f(t) = + (k + ) t –
t2
4
Clearly, f(t = 0) < 0 (so Disc. > 0)
 For no solution,
 
f 0 .......(1) and f  0 .......(2)
 2  2
must be satisfied simultaneously.
 3
Now, equation (1)  k
2

and equation (2)  k
2
  3   
 (1)  (2)  k  , . Ans.
 2 2 

184. Let a = eiA


b = ei2A
c = ei4A
Now abc = ei7A = 1
(cos A cos 2A cos 4A) [(1 + i tan A) (1 + i tan 2A) (1 + i tan 4A)] = 1 + 0i
(cos A cos 2A cos 4A) 1  iS1  S2  iS3  = 1 + 0i
1  iS1  S2  iS3  8 + 0i
 1 – S2 = 8  S2 = –7
Alternative :
2 
A = 2B = B=
7 7
7B =  ans 8B =  + B
tan 2B tan 4B + tan 4B tan 8B + tan 8B tan 2B
tan 2B tan 4B + tan B[tan 2B + tan 4B]
tan 2B  tan 4B tan 2B  tan 4B
tan 6B = 1  tan 2B tan 4B   tan B = tan 2B tan 4B = 1
tan B

(1  tan 2B)  sin 2B sin 4B   1 


(tan 2B + tan 4B) 1        1
tan B  cos 2B cos 4B   sin B cos B 

sin 6B
1
sin B cos B cos 2B cos 4B

continued product

8 sin 6B  
1 B  
sin 8B  7
8 sin B
1
 sin B
–8+1=–7  7 Ans.
FINAL STEP Page # 97
MATHEMATICS
x2
185. f (x) = Aex – 1 – Bx + ......(1)
2
where A = f ' (1); B = f (0)
f ' (x) = Aex – 1 – B + x ......(2)
Put x = 1 in (2)  f ' (1) = A – B + 1
A=A– B + 1
 B=1
from x = 0 in (1)
A
f (0) =
e
A A
B=  1=  A=e
e e
x2
Hence f (x) = – x +ex
2
x
Now f ' (x) = e + x – 1
decreasing in (– , 0) and increasing in (0, )
 minimum value occur at x = 0 and minimum value is f (0) = 1 Ans.

186. We have Y – y = m (X – x)
mX – Y + (y – mx) = 0

y  mx
Now, p = =x (given) y
1  m2
P (x, y)
 (y – mx)2 = x2 (1 + m2) (on squaring both sides)
y2 – 2xym = x2
M
dy y2  x 2
or = ...(1) x
dx 2 xy O
2
put y = t
dy dt
2y = ; substituting it in (1), we get
dx dx
dt dt 1
x = t – x2  – t=–x (Linear differential equation)
dx dx x
1
Now, I.F. = = integrating factor
x
1
x 
So, general solution is t · =  dx y

t x
 = – x + C y2 = – x2 + Cx x2 + y2 = Cx O (2, 0)
x
If x = 2, y = – 2  C = 4
So, x2 + y2 – 4x = 0, which is circle of centre (2, 0) and radius 2
 Area above the x-axis = 2 k = 2 Ans.

FINAL STEP Page # 98


MATHEMATICS

6

187. I=  ln ( 3  tan x ) dx  12 ln 3
0 I1

Using King in I1
 
6
 
6  1  3 tan x 
I1 =  ln ( 3  tan   x  dx
6 
 I1 =  ln  3

 dx
3  tan x 
0 0 

6
 4 
I1 =  ln  13  tan x  dx
0

 
 2I1 = (ln 4)  I1 = ln 4
6 12
 4
 I= ln   . Ans.
12 3

d 2 y dy
188. Given, 9 + f "(x) + f '(x) = x2 + f 2(x) or 9+  = x2 + y2
dx  dx

dy d2y d2y
As, P be the point of maxima of f(x) so = 0 and 2 < 0  x + y = 9 + 2 ]
2 2
dx dx dx
2 2
So, P(x, y) will lie inside the circle x + y = 9.
Hence, no tangent is possible. Ans.]

 f ( x ) 
1
1 2 2
189. Given,  f ( x ) dx = + dx ... (1)
0 3 0
Consider,

 f ( x ) 
1
2 2
I1 = dx
0
Put x2 = t 2x dx = dt

1
1
f (t ) 2 dt
=
2
 t
0
 Equation (1) becomes
1 1
f (t ) 2  2 1

 f ( t )  t  dt
t t f (t )
2 
2
dt + dt  =0
0
t 0
2 t
0

 f(t) = t
1 1
So, f  
4 2
1
So, reciprocal of f   = 2. Ans.
4
FINAL STEP Page # 99
MATHEMATICS
190. f ' (x0) = x03 = –1
 x0 = –1 x+y=0
Also, x0 + y0 = 0  y0 = 1
(x0 y0)
4
x
Now, y = f(x) = c
4

x 04
Also, y0 = c
4
1 3
1= c  c=
4 4
x4  3
 f(x) =  4f(x) – x4 = 3. Ans.
4

3 2 5
191. Given, f(x) = (x2 – 2x) · ln x – x  4x 
2 2
1 3
 f '(x) = (2x – 2) ln x + (x2 – 2x) · – 2 x   4 .
x 2
= 2( x – 1) ln x + (x – 2) – 3x + 4 = 2 (x – 1) ln x – 2(x – 1) = 2 (x – 1) · (ln x – 1)
So, f ' (x) = 0  x = 1, e.
+ – +
0 1 e
sign scheme of f '(x)
3 5
 f(x) has local maximum at x = 1.  flocal maximum (x = 1) = 0   4  = 1 + 4 = 5.
2 2
Note : Domain of f(x) = (0, ). Ans.

sin x
sin x
e t dt e  t dt
192. g(x) = e2x +   g(x) = e2x + e sin x  1 t2
0 cos 2 x  2 t sin x  t 2 0

king

g(0) = 1  f(1) = 0 sin x


e  t dt
g '(x) = 2e2x + sec x + e sin x · cosx  1 t2
g'(0) = 3 0

g (f (x)) = x
g f ( x )  · f '(x) = 1
g' f (1)  · f ' (1) = 1
1
g ' (0) · f '(1) = 1  f '(1) = .
3
sin x
e  t dt
g "(x) = 4e2x + sec x · tan x + sec x + esin x · cos x  1 t2
0
g(0) = 1
g'(0) = 3
g"(0) = 5. Ans.

FINAL STEP Page # 100


MATHEMATICS

193. Normal to plane x – y + z = 0 is n1  î  ĵ  k̂

Normal to plane x + y + z – 2 = 0 is n 2  î  ĵ  k̂

î ĵ k̂
 
Now, n1  n 2  1 1 1 =  2î  2k̂
1 1 1

D.r.'s of line of intersection of two given plane  (– 1, 0, 1)


Putting z = 0 in equation of both planes we get
x – y = 0 and x + y = 2
Solving these, x = 1 = y
 A point on line can be = (1, 1, 0)
 Equation of line of intersection in symmetric form is
x 1 y 1 z  0
  
1 0 1
Let foot of perpendicular from origin on the line be M then M = (1 – , 1, )
 drs of OM = 1– , 1, 
 OM is perpendicular to line of intersection.
1
 – 1 (1 – ) + 0 + 1 ·  = 0   =
2
1 1
 Dr's of OM =  , 1, 
2 2
 Required plane is farthest from origin
 OM must be normal to plane and plane should contain line of intersection (Point M too)
Equation of plane is
1 1 1 1
 x    1 y  1   z    0  x + 2y + z = 3
2 2 2 2
 (a + b + c + d)least = 7. Ans.
Aliter: Equation of any plane containing the line x – y + z = 0 and x + y + z – 2 = 0, is
(x – y + z) +  (x + y + z – 2) = 0 or (1 + ) x + ( – 1) y + (1 + ) z – 2 = 0 .......(1)

2 2
The distance of above plane from (0, 0, 0) is = 2 2 2
= 2
(1   )  (  1)  (1   ) 3  2  3

2
Now, we want  for which f() = , is maximum
32  2  3

(32  2  3) · 2  2 (6  2) 2 (  3)
 f '() = =
(32  2  3)2 (32  2  3) 2
So, f '() = 0 gives  = – 3, 0
+ – +
–3 0
(Maximum) (Minimum)
Sign Scheme of f '( )
 f() will be maximum for  = – 3.
So, putting  = – 3 in equation (1), we get – 2x – 4y – 2z + 6 = 0 or x + 2y + z – 3 = 0. Ans.

FINAL STEP Page # 101


MATHEMATICS
1 100
1
194.  [sec x ] dx   [sec1 x ] dx y
100 1 3 

sec 2 sec 3 1 sec 1 100 3


= 1 · dx   2 · dx   3 · dx   0 · dx  1 · dx 2
/2
100 sec 2 sec 3 1 sec1
1

x
sec2 sec3 – 1 O 1 sec 1

= [sec 2 – (– 100)] + 2 (sec 3 – sec 2 + 3 (– 1 – sec 3) + 0 + (100 – sec 1)


= 197 – (sec 1 + sec 2 + sec 3)
3
= 191 –  sec (i)
i 1
 p = 197. Ans.

(4, 4)

C2
(4, 2)

C1 C
195.
(–4, 0) (–2, 0) 0 ( r, 0) (4, 0)

C3
(4, –2)

(4, –4)

CC1 = CC2 = CC3 = r + 2


(r + 2)2 = (r – 4)2 + 4 = (r – 4)2 + 4
r2 + 4 = 4r = r2 + 16 – 8r + 4
12r = 16
4 p
r= = p + q = 7. Ans.
3 q

FINAL STEP Page # 102


MATHEMATICS
4  16  4 42 3
196. Given, x2 – 4x + 1 = 0  x = = =2± 3y
2 2 A

 From above figure, I (a, b) lies on line y = x. So b = a y=x

r m1  2  3
Also, from IBD, = sin 30°  BI = 2 [As, r = 1 (given)] I (a, b)
BI r=1
45° C
75°
D m2  2  3
 BI = 2 = a 2  a 2  4 = 2a2  a = 2 (0, 0)
B
30°
15°

 
x
So, I = 2 , 2 = (a, b) (Given)

Hence, (a2 + b4) =  2   2


2 4
= 2 + 4 = 6. Ans.

1
x i ( x 2012  1)
197. I2012 + i – Ii =  dx = 1
0
x 2012  1 i 1
1
x j  3 ( x 2013  1) 1
J2016 + j + Jj + 3 =  2013
dx =
0
x 1 j 4

1 1 1
a11 = , a22 = , a33 =
2 3 4
1 1 1
b11 = , b22 = , b33 =
5 6 7
–1
trace (A ) + det (B )–1

= (2 + 3 + 4) + (5 · 6 · 7) = 9 + 210 = 219 Ans.

1  cos 1  cos (1  cos x )  1  cos (1  cos x ) 2


198. l = Lim ·
x 0 1  cos (1  cos x ) 2 xa
2 8 4
1 1  cos (1  cos x )  (1  cos x )4 1  1  cos x  x
= Lim   · = Lim   ·
2 x 0  (1  cos x )2  xa 8 x 0  x   x a

1 x (8  a ) 1 1 ( 8 a )
= · = · Lim x
8 16 8 16 x  0

x 8 a
l = Lim
x 0 128
For existence of limit 8 – a  0  a  8
If a < 8 then l = 0
1
If a = 8, then l =
128
If a > 8 then l does not exist.
Hence, the existence & non-zero value a = 8. Ans.

FINAL STEP Page # 103


MATHEMATICS
199. Number of constant function = 4 (think!)  n
Total function = 45 = 1024 A B
1 a
5!
Number of onto from A to B = 240 11 | 1 | 1 | 1
   5
Grouping of five
2! · 1! · 1! · 1! · 3! 2 d b
element into 4
4
5! · 4! 3 c
There are to distributed in four children  = 240
2! · 3!
 Number of NOT ONTO = 1024 – 240 = 784  m.

200. Given sin x + sin y = 1


dy
cos x + cos y =0 .......(1)
dx
again
2
d 2 y  dy 
– sin x + cos y    sin y = 0
dx 2  dx 
2 2
d2y  dy   cos x 
or cos y 2 = sin x + sin y   = sin x + sin y  
dx  dx   cos y 

d2y
cos3y 2 2
2 = sin x cos y + sin y cos x
dx

d2y sin x (1  sin 2 y)  sin y (1  sin 2 x )


=
dx 2 (1  sin 2 y) 1  sin 2 y
Using sin x + sin y = 1

d2y
= Lim
 
sin x 1  (1  sin x ) 2  (1  sin x ) (1  sin 2 x )
dx 2 x 0
 
1  (1  sin x ) 2 1  (1  sin x ) 2

sin x (2 sin x  sin 2 x )  (1  sin x ) (1  sin 2 x ) sin 2 x (2  sin x )  1  sin 2 x  sin x  sin 3 x
= =
(2 sin x  sin 2 x ) 2 sin x  sin 2 x sin x  3 2 2  sin x  3 2
d2y sin 2 x  sin x  1
=
dx 2 sin x  3 2 2  cos x  3 2
d2y x sin 2 x  sin x  1
Lim x = ·
x 0 dx 2 sin x  3 2 2  sin x  3 2
3 3
for non-zero existence of limit  = (if  > then limit will be zero)
2 2
1
and L=
2 2
2
 3 2 2 
 = · =3 2    = 18 Ans.
L 2 1 L

FINAL STEP Page # 104


MATHEMATICS
201.
(i) Let x < 0
x
|y|= – | x – 60 |
4
x 3x
|y|=– + x – 60 = – 60
4 4
for x < 0, R.H.S. is always negative, which is not possible as | y |  0
(ii) Let 0  x  60
x
(60 – x) + | y | =
4
x 5x
|y|= + x – 60 = – 60
4 4
5x 240
Now, – 60  0  x  or x  48
4 5
5x  5x
Hence y = – 60 or + 60 for 48  x  60 [ y will not exist for x  [0, 48) ) ]
4 4
If x  60
x  3x
|y|= – (x – 60) = + 60
4 4
y
3x 3x
Now, 60 – 0   60 B
4 4
 60  x  80 A
15
C
x
For 60  x  80, (y will not exist if x > 80) O
48 60 80

3x 3x
y = 60 – or – 60. D
4 4
The given equation represents a kite as shown
(30) · (32)
A= = 480 Ans.
2

tan 1 3 tan 1 3

 ln 1  3 tan(tan    3  tan x  
202. I=
1
3  x ) dx =  ln 1  3

  dx
 1  3 tan x  
0 0

tan 1 3 tan 1 3
 1  3 tan x  9  3 tan x 
=  ln 
 1  3 tan x
 dx =

 ln (10)  ln (1  3 tan x) dx
0 0

tan 1 3
1
2I =  ln (10) dx  I= (tan 1 3) (ln 10) ]
0
2

FINAL STEP Page # 105


MATHEMATICS
203. Given, z +  = i .......(1)
and z2 + 2 = 1 .......(2)
 From (1), on squaring we get
z2 + 2 + 2z = – 1  1 + 2z = – 1 [Using (2)]
 zw = – 1 ........(3)
Now, let us consider a quadratic equation in x whose roots are z and .
 x2 – ix – 1 = 0

i 3 P(0,0)

2
i i 4 i 3 2
 x=  . Let x
2 2 i 3
z i 3
R  

2  i 3   2 
Q  

 2 

0 0 1

1 3 1 1 3 3 3
So, ar. (PQR) = 1 = 2  4  4   4 . Ans.
2 2 2  

 3 1
1
2 2

           
204. We have [(a  b)  ( b  c ) ( b  c )  c  a  c  a   (a  b)]
                 
= [a  b  b  c  c  a ,  a  b  b  c  c  a ,  a  b  b  c  c  a ]
      

        
Let a  b  l , b  c  m , c  a  n so that E = l  m  n ,  l  m  n ,  l  m  n 

E = V1 V 2 V 3  where
V1  l  m  n
V 2  l  m  n
V 3  l  m  n

1 1 1

= V1 V 2 V 3  = 1 1 1 l m n
1 1 1
 

1 1 1
         
=  1 1 1 [ a  b b  c c  a ] = 4 [ a b c ] 2  k [ a b c ]2
1 1 1
Hence k = 4 Ans.

FINAL STEP Page # 106


MATHEMATICS
x x
f (x) =  e sin( x  t ) dt =  e x  t sin( t ) dt
t
205. (usingKing)
0 0
x
f (x) = ex  e  t sin t dt
0

 x t  x
f ' (x) = ex · e–x sin x +   e sin t dt  e
 
0 
f ' (x) = sin x + f (x) ....(1)
f '' (x) = cos x + f ' (x) = cos x + sin x + f (x) [using (1)]
f '' (x) – f (x) = sin x + cos x ....(2)
 g (x) = sin x + cos x  g (x)   2 , 2   = [a , b]
 (a2 + b2) = 4. Ans.

206. Note that z = 0 is one root of the given equation, z4 + az3 + (12 + 9i) z2 + bz = 0
cancelling z, we get
z3 + az2 + (12 + 9i) z + b = 0 .......(1)
The 3 vertices of the square other than (0 + 0i) are u, iu, u (1 + i) (think!) Im (z) u (1 + i)

Now, u (ui) + (iu) (1 + i) u + [(1 + i) u] u = 12 + 9i (using theory of equation)


iu
u2 [i + i – 1 + 1 + i] = 12 + 9i u
3i u2 = 12 + 9i
O Re (z)
4  3i
u2 = = 3 – 4i  u = 3  4i = ± (2 – i)
i
 u = 2 – i or – 2 + i (both values of u will give the same result)
again, – a = u + ui + u (1 + i) = u [1 + i + 1 + i] = 2u (1 + i) = 2 (2 – i) (1 + i) = 2 (3 + i) = 6 + 2i
a = – 6 – 2i
Again (u) (ui) (1 + i) u = – b
– b = u3 (– 1 + i)
b = (2 – i)3 (1 – i) = [8 – i3 – 6i (2 – i)] (1 – i) = (8 + i – 12i – 6) (1 – i) = (2 – 11i) (1 – i)
b = 2 – 2i – 11i – 11 = – 9 – 13i
 b = – 9 – 13i
Hence, 2b – 13a = 2 (– 9 – 13i) – 13 (– 6 – 2i) = 60. Ans.

207. This problem can be viewed as


y
S= ( x  2) 2  1  ( x  1)2  22 B(–1,2)

S = | PA – PB | A(–2,1)
But | PA – PB |  AB
x
 | PA – PB |max. = AB = 2 =S –2 –1 P(x,0)

 S4 = 4 Ans.]

FINAL STEP Page # 107


MATHEMATICS
1
2 x  3x 2  4 x 3
208. = 2 x 4  x3  x 2
dx
0
Put (x2 + x3 + x4) = t  (2x + 3x2 + 4x3) dx = dt
3
dt
= 2 t
0

3
 
=  t  = 3  2 = 3 Ans.
 0

209. Since range contains exactly 3 distinct values.


This problem is equivalent to finding the number of ways in which 5 books can be divided into 3 children
so that each child gets atleast one book.
f
1 1
2 2
3 3
4 4
5 5

 5! 1 5! 1 
 1 ! 1! 3! 2 !  1 ! 2 ! 2! 2 !  3 ! = 1500. Ans.
5C
3
 

210. The equation f(x) = 0 has 3 distinct real roots , ,  where – 2 <  < – 1, 0 <  < 1 and 1 <  < 2
So, the equation f f ( x )   0 has 7 distinct real roots.
Y
+

(–1, 3) (2, 3)

f(x) = 
× × (0, 1) ×
x = –2 f(x) = 
×x =  × ×
X
x = –1 (0,0) x=1 x=2
(–2, –1)
(1, –1) f(x) = 
×
Graph of f(x) = x – 3x + 1, x  (– )
3
–

Hence, the equation f f ( x )   0 has 7 distinct real roots . Ans.

FINAL STEP Page # 108


MATHEMATICS

 f 
1
 1 
 
1
2
211. f(x) = x2 – ( t )  x 2  2 x f ( t ) dt = x2 –  x 2  dt   f 2 ( t )  2 xf ( t ) dt 
0  0 0 
1 1

 f (t ) dt  2x  f ( t ) dt
2
f(x) = – ........(1)
0 0

1
f '(x) = 0 – 2  f ( t ) dt = constant  f must be a linear function
0
Let f(x) = ax + b
1
 a = – 2  (at  b) dt ........(2)
0

1
and b = –  (at  b)2 dt ........(3)
0

1
 at 2 
From (2), a = – 2   bt 
 2 0

a 
 a = – 2   b  = – a – 2b
2 
2a = – 2b  a + b = 0
1
( t  1)3   1
From (3), – a =  a 
2
 1 = a  0    a = 3 and b = – 3
3   3
0

 f(x) = 3(x – 1)  f (3) = 3(3 – 1) = 6. Ans.

212. Clearly S denotes a region bounded by x = ± 1 and y = ± 1 y


i.e. square of side 2 on the complex plane. x= –1
Hence n(S) = 4 ....(1)
x
3 (–1,0) O (1,0)
now let w = (1  i)( x  iy)
4
3
(x  y)  i( x  y)
=
4
In order that w lies on S
– 1  Re (w)  1 and – 1  Im (w)  1
3 3
hence – 1  ( x  y)  1 and – 1  ( x  y)  1
4 4
4 4 4 4
–x–y and –  x + y 
3 3 3 3
Consider, x – y = 4/3 ; x – y = – 4/3
x + y = 4/3 ; x + y = – 4/3
4 4 1 32
Area EFGH = 2 · ·2· · =
3 3 2 9
FINAL STEP Page # 109
MATHEMATICS
Area of small triangles which is outside the region 'S' is
1 2 1 4
=4· · · =
2 3 3 9
32 4 28
n(A) =  =
9 9 9
n (A) 28 1 7
 Probability = = · =  m  7 + 9 = 16 Ans.
n (S) 9 4 9 n

1
213. Area = ab
2
AD : y = x + 3
BE : y = 2x + 4
solve G(–1, 2)
acute angle  between the medians is A

m1  m 2
tan  = 1  m m b/2

1 =1
1 2 (–1,2)

m
F  E
2 1 1 G

tan  =  tan  = 2
1 2 3 m 2=   – 
b/2
now (180 – ) + 90° +  +  = 360° 90°

B C
  =  +  – 90° a/2 D a/2
cot  = – tan( + )
2b 2a
tan   tan  
a b 2( a 2  b 2 )
–3= or –3=  9=
1  tan  tan  2b 2a ab
1 ·
a b
1
9ab = 2 × 3600  ab = 400
2
 Area = 400 sq. units

214. |z – (4 + 3i) | = r y
For one solution B
z=4+0·i
C
 r=3
(4, 3)
Now,
| z – (4 + 3i) | = 3
x
Maximum value of z = OC + CB O A
= 5 + 3 = 8. Ans. (4, 0)

FINAL STEP Page # 110


MATHEMATICS

2
215. Let  sin x · f (x ) dx = c
0

c sin x
 f(x) = sin3x +

  
2 2 2
 c sin x  c
Now, c =  sin x  sin 3 x   dx =  sin x dx 
4
 sin
2
x dx
0    0
 0

  
3 c  2 4 3 2

c=  ·   sin x dx  and  sin 2 x dx  
16  4 0 16 0
4
 

3c 3 
  c=
4 16 4
sin x 
 f(x) = sin3x + ·
 4
sin x
f(x) = sin3 x +
4
  
2 2 2 2 1 11
1
 =  f ( x ) dx =  sin
3
x dx   sin x dx =  =
40 3 4 12
0 0

 12 = 11. Ans.

216. Since, C1 and C2 touches each other.


ln x
 = x2 .......(1)
x
 ln x = x3 .......(2)
differentiate w.r.t. x equation (1)
1 – ln x
2
 2x  1 – x3 = 2x3
x y
y = x2
1 y = lnx
1 x
 1 3
 3x3 = 1  x =    3  3 O
1/3
1e e
x
 3 
1
1 1 1
From (2), ln (3) =  ·  ln (3) = – 1.   = , x  e3
3 3 3e
1 1
1 1

   
e3 2 e3
x ln x 1 3 e3 1 e3
Area =  dx   dx = x 0  lnx 2 1
0
3 e 1
x 9e 2

1 1 1
=   sq. units. Ans.
9 18 18

FINAL STEP Page # 111


MATHEMATICS
217. Let z = a + ib are | z | = r
| 2z + 3i|2 = | z2 |2 = | z |4
r4 = (2z + 3i) ( 2 z  3i )
2
r4 = 4zz  6zi  6 zi  9i
2
= 4r  6i( z  z )  9
= 4r2 – 12i (Im z)i + 9 [Using z  z  2  Im z (Im z = b) ]
2
= 4r + 12b + 9
12b = r4 – 4r2 – 9
12(Im z) = r4 – 4r2 – 9
But –|z|  Im z  |z|
–r  b  r
–12r  12b  12r
–12r  r4 – 4r2 – 9  12r
Also r  0
How r4 – 4r2 – 9  12r
r4 – 4r2 – 12r – 9  0 .....(1)
(r – 3) is a factor of (1)
 r3(r – 3) + 3r2 (r – 3) + 5r (r – 3) + 3 (r – 3)
(r – 3) (r 3  3r 2  5r  3)  0

Always  ve

 r  |z|max. = 3  M = 3
Again
– 12r  r4 – 4r2 – 9
r4 – 4r2 + 12r – 9  0
(r – 1) is a factor.
r (r – 1) + r2 (r – 1) – 3r (r – 1) + 9 (r – 1)  0
3

(r – 1) (r 3  r 2  3r  9)  0

 ve for r 

 r  |z|min. = 1  m=1


 M + m = 3 + 1 = 4. Ans.

k  t2 Y
218. h = t and =3 P
2 (t, t2)
y=3
k + t2 = 6 (Image on P in y = 3)
k+h =62 .....(1) P' (h, k) X
O
Again, k = 6 – T2 where h = T
h+T=4
k = 6 – (4 – h)2
k = 6 – (16 + h2 – 8h) Y
k + 10 = 8h – h2 (Image of P' in x = 2) (t, 6 – t2) (h, k)
y = –x2 + 8x – 10 P' Q
 (b, c)  (8, –10) O
 b – c = 18. Ans. x=2

FINAL STEP Page # 112


MATHEMATICS
  
219. Let bc  p
Now,  
          
   
a  b  p = a · p  b  b · p a = a b c b = b

1 1
      1
a · a a · b a · c 2 3
1 1 5
 
  
 2 = a b c 2
= b · a b · b b · c = 1 =
     2 2 9
c·a c·b c·c 1 1
1
3 2
Hence, 92 = 5. Ans.

220. Given, f(x) = x2 – ax + 2a  f '(x) at (2,4) = 2x – a = 4 – a


1
 slope of normal =
a4
 Equation of normal at P (2, 4) is
1
(y – 4) = (x – 2)
a4
Put y = 0, we get x = (18 – 4a)
 2 
Put x = 0, we get y =  4  
 a 4
Now, Area = 2 (Given)
1  2 
 (18 – 4a)  4   =±2  (2a – 9)2 = 1
2  a 4
 2a – 9 = ± 1  2a = 9 ± 1
 a = 5, 4
But a = 4 (reject)
So, a = 5. Ans.

221.  
The equation of normal to y2 = 16x at N 12, 8 3 is y  3x = 20 3 .
y-axis (12, 8 3)

p
90°

x-axis
V(0,0) S(4,0)

20 3  4 3
 Length of perpendicular from S(4, 0) on y  3x = 20 3 , is = =8 3.
2
 p= 256  192 = 64 = 8. Ans.

FINAL STEP Page # 113


MATHEMATICS
222. The given equality can be written as
z13  z 32  z 33  z l z 2 z 3 = 0
or – 4z1 z2 z3 = z13  z 32  z 33 – 3z1 z2 z3


= (z1 + z2 + z3) z12  z 22  z 32  z1z 2  z 2 z 3  z 3z1 
Setting z = z1 + z2 + z3, yields
z3 – 3z(z1z2 + z2z3 + z3z1) = –4z1 z2 z3
This is equivalent to
 1 1 1 
z3 = z l z 2 z 3 3z     4
  z1 z 2 z 3  
The last relation can be written as

    2
z3 = z l z 2 z 3 3z z1  z 2  z 3  4 , i.e. z3 = z1 z2 z3 3 | z | 4 . 
Taking the absolute values of both sides yields | z |3 = | 3 | z |2 – 4 |.
2 2
If | z |  , then | z |3 – 3 | z |2 + 4 = 0, implying | z | = 2. If | z | < , then | z |3 + 3| z |2 – 4 = 0,
3 3
giving | z | = 1, as needed.
3 3 3
Aliter: It is not difficult to see that z1  z 2  z 3 = 1. By using the algebraic identity
(u + v) (v + w) (w + u) = (u + v + w) (uv + vw + wu) – uvw
for u = z13 , v = z 32 , w = z 33 , it follows that

z 3
1      
 z 32 z32  z 33 z 33  z13 = z13  z32  z33 z13z 32  z32 z 33  z 33z13  z13z 32 z 33

3 3 3 3
 3 3  1
 1 1 3 3 3
= z1 z 2 z 3 z1  z 2  z 3  3  3  3   z1 z 2 z 3
 z1 z 2 z 3 
3 3 3 3
 3 3  3
 3 3 3 3 3
= z1 z 2 z 3 z1  z 2  z 3  z1  z 2  z 3   z1 z 2 z 3 = z13z32 z33  z13z32 z33  0
 
Suppose that z13  z 32  0 . Then z1 + z2 = 0 or z12  z1z 2  z32  0 implying

z12  z 22 = –2z z or z12  z 22 = z z


1 2 1 2

On the other hand, from the given relation nit follows that z 33 = –z1 z2 z3, yielding z 32  z1z 2
We have
 
| z1  z 2  z 3 |2 = (z1  z 2  z 3 ) 1  1  1 
z z z 3 
 1 2

z z  z z  z z 
= 3   1  2    1  3    2  3 
 z 2 z1   z 3 z 2   z 3 z1 

z12  z 22 z 22  z1z 2 z 32  z1z 2 z12  z 22


= 3   = 3  .
z1z 2 z 2z 3 z 3z1 z1z 2

This leads to |z1 + z2 + z3|2 = 1 if z12  z 22 = – 2z1z2 and |z1 + z2 + z3|2 = 4 if z12  z 22 = z1z2 .
The conclusion follows.

FINAL STEP Page # 114


MATHEMATICS
223. Let x1, x2, x3, x4 be the real roots of the polynomial f.
Then f = (x – x1) (x – x2) (x – x3) (x – x4)
and | f(i) | = 1  x12 · 1  x 22 · 1  x 32 · 1  x 24
Because | f(i) | = 1, we deduce that x1 = x2 = x3 = x4= 0 and consequently a = b = c = d = 0.
Hence, (a + b + c + d) = 0. ]

A (7, 6, 2) L1
x7 y6 z2
224. L1 :   = r1
3 2 4
x 5 y3 z4 B (5, 3, 4) C
L2 :   = r2
2 1 3 L2
Any point C on L2 is (5 + 2r2, 3 + r2, 4 + 3r2)
L3
 Direction ratio of L3 line is (5 + 2r2 – 7, 3 + r2– 6, 4 + 3r2 – 2)
 (2r – 2, r – 3, 3r + 2) and it is perpendicular to vector 2î  2ˆj  k̂ .
2 2 2
 (2r2 – 2) × 2 + (r2 – 3) × (–2) + (3r2 + 2) × (– 1) = 0  r2 = 0
 C is (5, 3, 4)
 | AC | = (7  5) 2  (6  3)2  (2  4)2 = 17

  | AC |  = 4. Ans.

y
y=5–x

y=7
x (0,1) y=x–5
–x
y=7
225.
x
O(0,0) (5,0)

From above graph,  = number of points of discontinities = 0


 = number of points of non-derivability = 2
Hence ( + ) = 0 + 2 = 2. Ans.

FINAL STEP Page # 115

You might also like